MCAT Discrete Practice Questions

Réussis tes devoirs et examens dès maintenant avec Quizwiz!

Which conclusion is best supported by the outcome of Study 2? A.Synchronous activity increases group polarization. B.Synchronous activity increases conformity. C.Asynchronous activity increases cognitive dissonance. D.Asynchronous activity increases social loafing.

he answer to this question is B because the results of Study 2 supported the hypothesis that synchronous actions would make participants more prone to comply with prompts to act aggressively (i.e., conform to the confederate's suggestion to choose the "noise blast").

Which protein properties allow UV spectroscopy to be used as a method of determining concentration? A. Proteins have partially planar characteristics in peptide bonds B. Globular proteins cause scattering of light C. Proteins contain aromatic groups in certain amino acids D. All organic macromolecules can be assessed with UV spectroscopy

UV spectroscopy is best used with conjugated systems of double bonds. While the double bond in the peptide bond does display resonance, this is not adequate for UV absorption. However, aromatic systems are conjugated, and phenylalanine, tyrosine, tryptophan all contain aromatic ring structures.

Which of the following species is represented by the electron configuration 1s^2 2s^2 2p^6 3s^2 sp^6 4s^1 3d^5 I. Cr 2. Mn + 3. Fe^2+ A. I only B. I and II only C. II and III only D. I, II, and III only

When dealing with ions, you cannot directly approach electronic configurations based on electrons they currently hold. First examine the neutral atom's configuration, and then determine while electrons are removed. Due to the stability of the half-filled d-orbitals, neutral chromium assumes the electron configuration of [Ar] 4s^1 3d^5. Mn must lose one electron from its initial configuration to become the Mn+ cation. That electron would come from the 4s subshell according to the rule that the first electron removed comes from the highest-energy shell. Fe must lose tow electrons to become Fe^2+. They'll both be lost from he same orbital; the only way Fe^2+ could hold the configuration in the question stem would be if one d-electron and one s-electron where lost together.

When glucose is in a straight-chain formulation, it: A. is an aldoketose B. is a pentose C. has five chiral carbons D. is one of a group of 16 steroisomers

A

In the chromatography of the reaction mixture, water absorbed on cellulose functioned as the stationary phase. What was the principal factor determining the migration of individual components in the sample? A.Hydrogen bonding B.Solute concentration C.Stationary phase concentration D.Thickness of paper

A This is an Organic Chemistry question that falls under the content category "Separation and purification methods." The answer to this question is A because the relative amount of hydrogen bonding to the stationary phase will determine the relative rate of migration of the various components in the sample. It is a Scientific Reasoning and Problem Solving question because you are asked to evaluate an argument about causes and consequences (the primary reason for chemical separation during a chromatographic separation on cellulose).

During an interview with a schizophrenic patient, a psychiatrist notices that the patient keeps repeating what the psychiatrist says. This phenomenon is known as: A. echolalia B. echopraxia C. loosening of associations D. neologisms

A. Echolalia is an involuntary repetition of other's words and utterances, and may be seen in schizophrenia. Echopraxia, choice (B), is imitation of other's actions. Loosening of associations, choice (C), is a type of disordered thought in which the patient moves between remotely related ideas. Neologisms, choice (D), are newly invented words.

All of the following are key processes in the production of energy in the mitochondrion Except: A. glycolysis B. the citric acid cycle C. electron transport chain D. oxidative phosphorylation

A. Glycolysis is important in energy production, but it occurs in the cytoplasm, not mitochondria. The other choices can be eliminated since they all occur in the mitochondria and are therefore incorrect.

Which of the following would decrease the likelihood of a bystander lending aid to a victim? A. Increasing the number of people in the room B. Increasing the degree of danger experienced by the victim C. Making the victim an acquaintance of a stranger D. Being alone in the room with the victim.

A. It has been observed that increasing the number of bystanders decreases the likelihood that any of them will aid a victim. Increasing the degree of danger experienced by the victim, choice (B), making the victim an acquaintance instead of a stranger, choice (C), and being alone in the room with the victim, choice (D), would increase the likelihood that the bystander would help the victim.

Suppose an atom fills its orbitals as shown: up/down up up up 3s all 3p Such an electron configuration most clearly illustrates which of the following law of atomic physics? A. Hund's rule B. Heisenberg uncertainty principle C.Bohr model D. Rutherford model

A. The MCAT covers the topics in this chapter qualitatively more often than quantitatively. It is critical to be able to distinguish the fundamental principles that determine electron organization, which are usually known by the names of the scientists who discovered or postulated them. The Heisenberg uncertainty principle, choice (B), refers to the inability to know the momentum and position of a single electron simultaneously. The Bohr model, choice (C), was an early attempt to describe the behavior of the single electron in a hydrogen atom. The Ruthordford model, choice (D), describes a dense, positively charge nucleus. The element shown here, phosphorus, is often used to demonstrate Hund's rule because it contains a half-filled p subshell. Hund's rule explains that electrons fill empty orbitals first before doubling up electrons in the same orbital.

According to the developed chromatography plate shown below, what is the approximate Rf value of aspartic acid? A.0.20 B.0.50 C.5 D.10

A. This is an Organic Chemistry question that falls under the content category "Separation and purification methods." The answer to this question is A (0.20) because Rf is the ratio of the distance travelled by the analyte relative to the solvent front during a chromatographic separation. Aspartic acid travelled two units, while the solvent front travelled ten units, giving an Rf of 2/10 = 0.20 for aspartic acid. It is a Data-based and Statistical Reasoning question because you are asked to analyze and interpret data presented in a figure to draw a conclusion.

The activity of an enzyme is measured at several different substrate concentrations, and the data are shown in the table below: a section of the table: [S](mM) (vmmol/sec) 0.1 17 0.5 50 1 67 Km for this enzyme is approximately: A. 0.5 B. 1 C. 10 D. 50

A. While the equation given in the text are useful, recognizing relationships is even more important. You can see that as substrate concentration increases significantly, there is only a small change in rate. This occurs as we approch Vmax because the Vmax is near 100 mmol/min, Vmax/2 equals 50 mmol/min. The substrate concentration giving this rate is 0.5 mM and corresponds to Km; therefore, choice (A) is correct.

Which of the following is an example of a Gesellschaft? A. A large corporation B. A small rural neighborhood C. Members of the same family D. An ethnic enclave in a large city

A. A Gesellschaft (society) is one in which individuals are working toward the same goal, such as a company or a country. Gemeinschaften (communities), on the other hand, are those that are bonded by beliefs, ancestry, or geography.

Epilepsy may result in motor seizures due to massive synchronous firing of neurons in a small area of the cerebral cortex (the epileptic focus). Excitation spreads from the focus, involving an increasingly larger area of the cortex. A drug for the treatment of epilepsy would be most effective if it caused which of the following changes in the epileptic focus? A.An increase in the neuron-firing threshold B.An increase in extracellular Na+ concentration C.A decrease in axon-membrane permeability to negative ions D.A decrease in the length of the depolarization stage

A. A drug that would best treat epilepsy, as described, should reduce the activity of the neurons in the epileptic focus, which are excitatory neurons based on their effect throughout the cortex. The answer to this question is A because increasing the threshold required to generate an action potential would decrease the chance that individual neurons would fire, thus reducing the overall amount of excitation that spreads from the epileptic focus throughout the cortex.

Which of the following factors determine an enzyme's specificity? A. The three dimensional shape of the active site B. The Michaelis constant C. The type of cofactor required for the enzyme to be active D. The prosthetic group on the enzyme

A. An enzyme's specificity is determined by the three dimensional shape of its active sites. Regardless of which explanation for enzyme specificity we are discussing (lock and key or induced fit), the active site determines which substrate the enzyme will react with.

Which of the following characteristics is NOT attributed to antibodies? A. Antibodies bind to more than one distinct antigen B. Antibodies label antigens for targeting by other immune cells C.Antibodies can cause agglutination by interaction with antigen D. Antibodies have two heavy chains and two light chains

A. Antibodies are specific to a single antigen. Each B-cell produces a single type of antibody with a constant region that is specific to the host and a variable region that is specific to the antigen.

Carbon and silicon are the basis of biological life and synthetic computing, respectively. While these elements share many chemical properties, which of the following best describes a differences between the two elements? A. Carbon has a smaller atomic radius than silicon B. Silicon has a smaller atomic radius than carbon C. Carbon has fewer valence electrons than silicon D. Silicon has fewer valence electrons than carbon

A. As one moves from top to bottom in a group (column), extra electron shells accumulate, despite the fact that the valence configurations remain identical. These extra electron shells provide shielding between the positive nucleus and the outermost electrons, decreasing the electrostatic attraction and increasing the atomic radius. Because carbon and silicon are in the same group, and silicon is further down in the group, silicon will have a larger atomic radius because of its extra electron shell.

Consider a reaction catalyzed by enzyme A with a Km value of 5x10^-6M and Vmax of 20 mol/min At a concentration of 5x10^-6 m substrate the rate of the reaction will be A. 10 B. 15 C. 20 D. 30 ----mmol/min

A. As with the last question, relationships are important. At a concentration of 5x10-6 M, enzyme A is working at one-half of it's Vmax because the concentration is equal to the Km of the enzyme. Therefore, one-half of 20 mmol/min is 10 mmol/min, which corresponds to choice (A).

Infrequently, the deletion of one or more nucleotides occurs in the replication of a cell's genome. In mutations in which three base pairs are deleted, the mutated gene codes for a relatively normal protein. A reasonable explanation for this observation is that: A. removal of a multiple of three nucleotides retains the original reading frame B. amino-acid codons often are flexible in their size C. translation of unmutated mRNA at the ribosome is successful only one-third of the time. D. Most mutations of DNA have little effect on cellular function

A. Codons consist of three base pairs. Deletion of one or two base pairs will change the reading frame for the rest of the gene, probably resulting in a completely nonfunctional protein after the paint of deletion. Deletion of three base pairs, however, will delete one entire codon and retain the original reading frame. The result in this case is that one amino acid is missing from the protein structure, but the rest of the protein is exactly the same as normal. Loss of one amino acid is a much more stable change than changing the sequence of a large portion of a protein, so protein function will probably be retained.

A woman comes tot he doctor with a two-week history of complete paralysis of her left arm. She has had no injury to the extremity, and full neurological workup fails to demonstrate any underlying cause. She seems surprisingly unconcerned about the paralysis, and seems more worried about an argument she had one month ago in which she hit her daughter. Based on this information, the woman's most likely diagnosis is: A. conversion disorder B. generalized anxiety disorder C. illness anxiety disorder D. histrionic personality disorder

A. Conversion disorder is marked by a motor or sensory symptom in the absence of an underlying physical or neurological cause. It is associated with an inciting event that, in this case, may have been the argument with her daughter. Her lack of concern over the deficit is refereed to as la belle indifference.

Which of the following statements is true regarding the separation of proteins via gel electrophoresis is INCORRECT? A. Decreasing the percent of polyacrylamide makes the gel less dense and decreases the ability to resolve small distances between two proteins with similar molecular weights. B. Proteins with smaller molecular weights travel more quickly than those with higher molecular weights when electric current is applied C. Proteins migrate to the positive electrode of the gel apparatus because they become negatively charged during their preparation for electrophoresis D. Proteins travel in denatured states through the gel

A. Decreasing the percent of the polyacrylamide (and the density) will increase your resolution capacity, and proteins with similar molecular weight will be seperated by a greater distance (choice A is false, and the correct answer choice) Proteins with smaller molecular weights travel more quickly than do proteins with larger molecular weights (choice B). The proteins are negatively charged because during preparation they are denatured (choice D) In a detergent that confers negative charge upon them, opposites attract. (choice C).

Which of these is most likely to be preserved when a protein is denatured? A. Primary structure B. Secondary structure C. Tertiary structure D. Quaternary structure

A. Denaturing a protein results in the loss of three-dimensional structure and function. Because the denaturation process does not normally result in breaking the peptide chain, the primary structure should be conserved. All of the other levels of structure can be disrupted.

Clathrin, a substance that aggregates on the cytoplasmic side of cell membranes, is responsible for the coordinated pinching off of membrane in receptor-mediated endocytosis. A lipid-soluble toxin that inactivates clathrin would be associated with: A. reduced delivery of polypeptide hormones to endosomes B. increased secretion of hormone into the extracellular fluid C. increased protein production on the rough endoplasmic reticulum D. in increase in ATP consumption

A. Endocytosis is the process by which the cell internalizes receptor-ligand complexes from the cell surface, such as polypeptide hormones bound to their receptor. At the cell surface, the receptor-lingand complexes cluster in clathrin-coated pits and pinch off the vesicles that join the acidic compartments known as endosomes.

Enzymes increase the rate of reaction by: A.decreasing the activation energy B. decreasing the overall free energy change of the reaction C. increasing the activation energy D. increasing the overall free energy change of the reaction

A. Enzymes increase the rate of a reaction by decreasing the activation energy. They do not affect the overall free energy , (change in G) of the reaction.

A 42-year-old woman has always been extremely neat and tidy. She works as a secretary and stays long after normal working hours to check the punctuation and spelling of letters she prepared during the day. Her boss referred her for counseling after she repeatedly got into fights with coworkers. "They don't take the job to heart,: she says. "They just joke around all day." The most likely preliminary diagnosis for this patient is: A. obsessive-compulsive personality disorder B. antisocial personality disorder C. narcissistic personality disorder D. borderline personality disorder

A. Focusing on details, loving routine, having a sense that there is only one right way to do things, and lack of humor suggests an obsessive-compulsive personality disorder.

The swimming times for all members of a swim team are tracked over a six-month period team-only practices, and at public meets. For 14 of the 16 members, top times were clocked at the meets. What social phenomenon does this evidence support? A. social facilitation B. peer pressure C. identification D. group polarization

A. For 14 out of the 16 members, the record times were obtained during public meets. The fact that the team members performed better when in front of a crowd supports the notion of social facilitation.

Which of the following is a true statement regarding fungi? A. Gametes may join together and exist in a dikaryon form until fusion of the nuclei occurs B. Production of endospores allows fungi to survive during times of nonideal environmental conditions C.During budding, fungi release hapliod hyphae with the intent of finding other hypae of the same species for sexual reproduction D. Hyphae that are specialized to digest and absorb nutrients are called fruiting bodies.

A. Gamets may join without immediate fusion of nuclei in fungi. The result is a cell with two nuclei, called a dikaryon. Fungi produce spores (not endospores, like bacteria for help with survival (B is wrong). Budding is an asexual process, not sexual (C is wrong). The hyphae that digest nutrients are called haustoria, not fruiting bodies. (D is wrong).

Which of these amino acids has a side chain that can become ionized in cells? A. Histidine B. Leucine C. Proline D. Threonine

A. Histidine has an ionizable side chain: its imidazole rings has a nitrogen aom that can be pronated. None of the remaining answers have ionizable atoms in their side chains.

How much net ATP would be produced if three molecules of glucose were used during anaerobic cellular respiration? A. 6 B.12 C.24 D. 36

A. In an anaerobic environment only glycolysis can occur. This process produces a net of 2 ATP per glucose, so three glucose molecules would produce 6 ATP total.

Which of the following is/are products of both glycolysis and the electron transport chain? I. ATP II. NADH III. Oxygen IV. FADH2 A. I B. I and III C. II and IV D. I, II, and III

A. Item I is true. ATP is produced in both the glycolysis and the electron transport chain. Item II is false: NADH is only produced in glycolysis. Items III and IV are false: oxygen and FADH2 are not produced in either of these processes.

Which of the following statements about enzyme kinetics is FALSE? A. In increase in the substrate concentration (at constant enzyme concentration) leads to proportional increases in the rate of the reaction B. Most enzymes operating in the human body work best at a temperature of 37 degrees C C. An enzyme - substrate complex can either from a product or dissociate back into the enzyme and substrate D. Maximal activity of many human enzymes occures around pH 7.4

A. Most enzymes in the human body operate at maximal activity around a temperature of 37 degrees C and a pH of 7.4, which is the pH of most body fluids. In addition, as characterized by the Michaelis-Menten equation, enzymes from an enzyme-substrate complex, which can either dissociate back into the enzyme and substrate or for a product. So far, we can eliminate choices (B) and (C) and (D), so lets check choice (A). An increase in the substrate concentration, leads to a proportional increase in the rate of the reaction only initially. However, once most of the active sites are occupied, the reaction rate levels off, regardless of further increases in substrate concentration. At high concentration of substrate, the reaction rate approaches its maximal velocity and is no longer changed by further increases in substrate concentration.

All of the following are true statements regarding the Krebs cycle EXCEPT: A. oxygen is directly required for it to occur B. NAD+ and FAD get reduced during the cycle C. two GTP are produced per glucose molecule D. in eukaryotic cells it occurs in the matrix of the mitochondria

A. Oxygen is required to run the electron transport chain during which NADH and FADH2 are oxidized. If oxygen is unavailable and the electron transport chain cannot run, there is no available NAD+ or FAD to participate in the Krebs cycle, thus we say that oxygen is only indirectly required for the Krebs cycle to occur (choice A is false and the correct answer). Choice B is true. Two GTP molecules per glucose are produced. and in eukaryotes, the Krebs cycle does occur in the matrix of the mitochondria.

Adult prison systems may attempt to change the behavior of inmates through all of the following mechanisms of socialization EXCEPT: A. primary socialization. B. secondary socialization. C. anticipatory socialization. D. resocialization.

A. Primary socialization is the teaching of acceptable actions and attitudes during childhood, which would occur too early to be part of the adult prison system. Re-socialization, choice (D), is the process by which one changes behaviors by discarding old routines and patterns and transitions to new behaviors necessary for a life change. The prison environment is designed to change bad behavior into desired behavior, making this an incorrect choice. When entering prison, an inmate must also undergo secondary socialization, choice (B), learning the rules of the specific social environment of the prison. Finally, if the inmate is not incarcerated for life, attempts at anticipatory socialization, choice (C), must be made before releasing the inmate in preparation for life outside of the prison.

Stockholm Syndrome is a phenomenon in which a victim of a kidnapping or hostage situation may begin to identify with or even feel affection for his or her captors. A psychoanalyst might explain Stockholm Syndrome by citing which of the following defense mechanisms? A. Reaction formation B. Regression C. Projection D. Displacement

A. Reaction formation is a defense mechanism that converts unwanted feelings into their exact opposite. A psycho-dynamic theorist would say that the terror and hatred one feels toward his or her captor might be unconsciously turned into affection in an effort to reduce the stress of the situation.

Antimycin is used as a piscicide (fish poison) because is inhibits Complex III of the electron transport chain. Blocking the flow of electrons through complex III will produce which of the following effects? I. Complex I (NADH dehydrogenase) will persist in a reduced state II. Complex IV (cytochrome C oxidase) will persist in a reduced state III. /Oxygen consumption will be decreased. A. Statements I and III only B. Statements II and III only C. Statements I, II, and III D. Statement III only

A. Reduced electron carriers like NADH and FADH2 transport electrons from the reaction of glycolysis, the pyruvate dehydrogenase complex, and the Kreb's cycle to the electron transport chain. Electrons are then relayed through the various proteins of the chain and finally to oxygen, which is reduced to water. Inhibiting any part of the electron chain will halt the transfer of electrons. Statement I is true: blocking electron transport at Complex III will result in a build-up of electrons at proteins earlier in the chain. Because these earlier proteins, including Complex I, cannot pass their electrons off, they will persist in the reduced state ("Gain of electrons is reduction"). Statement II, is false. Proteins later in the chain can release their electrons, but cannot replenish them. These later proteins, including Complex IV, will persist in the oxidized state ("Loss of electrons is an Oxidation"). Electrons meeting a road block at complex III will never reach oxygen. Thus oxygen consumption (reduction) will be decreased. It is important to note that inhibiting the electron transport chain diminishes the proton gradient and compromises ATP synthesis. This is why antimycin is so toxic to cells.

Which of the following this the correct electron configuration for Zn^2+? A. 1s^2 2s^2 2p^6 3s^2 3p^6 4s^0 3d^10 B. 1s^2 2s^2 2p^6 3s^2 3p^6 4s^2 3d^8 C. 1s^2 2s^2 2p6^3 3s^2 3p^6 4s^2 3d^10 D. 1s^2 2s^2 2p^6 3s^2 3p^6 4s^0 3d^8

A. Remember that when electrons are removed from an element, forming a cation, they will be removed from the subshell with the highest n valuefirst. Zn^0 has 30 electrons, so it would have an electron configuration of 1s^2 2s^2 2p^6 3s^2 3p^6 4s^2 3d^10 The 4s subshell has the highest princiuple quantum number, so it is emptied first, forming 1s^2 2s^2 2p^6 3s^2 3p^6 4s^0 3d^10 Choice (B) implies that electrons are pulled out of the d orbital, choice (C) presents the configuration of the uncharged zinc atom, and choice (D) shows the configuration that would exist if four electrons where removed

Which of the following statements represents the affective component of an attitude? A. "I love action movies" B. "I'm going to see a new action movie at the theater." C. "Action movies are much better than comedies." D. "Tomorrow, I'm going to rent an action movie."

A. The affective component of attitude consists of feelings and emotions toward something.

The evolutionary role of emotions had been used as support for which model(s) of emotional expression? A. The basic model only B. The social construction model only C. Both the basic model and social construction model D. Neither the basic model nor the social construction model

A. The basic model of emotions, as proposed by Charles Darwin, states that emotions serve an evolutionary purpose, and this are similar across cultures. The seven universal emotions have also been used as support for this theory. The social construction model states that emotions are always a product of the current social situation and does not posit any biological basis for emotions, implying a lack of a role for emotions in evolution.

It is known that the developing frog embryo requires greater protein production than the adult organism. If cells from a developing frog embryo and from a mature frog were examined, would the investigator find the greater rate of translation in cells of the embryo or of the adult? A. The embryo, because a developing organism requires a higher rate of translation than does an adult B. The embryo, because ribosomal production is not yet under regulatory control by DNA C. The adult, because ribosomal production is more efficient in a mature organism D. The adult, because a mature organism has more complex metabolic requirements.

A. The question state that embryos require greater protein production. Thus, by definition, an embryo requires a greater rate of translation, the process through which the ribosome creates a protein by reading an mRNA transcript.

How many total electrons are in a ^133Cs cation? A. 54 B. 55 C. 78 D. 132

A. The quickest way to solve this problem is to use the periodic table and find out how many protons are in Cs atoms; there are 55. Neutral Cs atoms would also have 55 electrons. A stable Cs cation will have a single positive charge because it has one unpaired s-electron. This translates to one few electron than the number of protons, or 54 electrons.

Which ion channels are responsible for maintaining the resting membrane potential? A. Ungated channels B. Voltage-gated channels C. Ligand-gated channels D. No ion channels are involved in maintenance of the resting membrane potential.

A. The resting membrane potential is displayed by cells that are not actively involved in signal transduction. Ungated or "leak" channels permit limited free flow of ions, while the sodium-potassium pump is also active and corrects for this leakage. Ligand-gated and voltage gated channels are involved in cell signaling and in the pacemaker potentials of certain cells, but cause deviation from - not maintance of - the resting membrane potential.

What property of protein-digesting enzymes allows for a sequence to be determined without fully degrading the protein? A.Selectivity B. Sensitivity C. Turnover D. Inhibition

A. The selective cleavage of proteins by digestive enzymes allows fragments of different lengths with known amino acid endpoints to be created. By cleaving the protein with several different enzymes, a basic outline of the amino acid sequence can be created.

Some studies suggest that in patient's with Alzheimer's disease, there is a defect in the way the spindle apparatus attaches to the kinetochore fibers. At which stage of mitotic division would one first expect to be able to visualize this problem? A. Prophase B. Metaphase C. Anaphase D. Telophase

A. The spindle apparatus first interacts with the kinetochore fibers near the end of prophase. While the spindle apparatus aligns the chromosomes at the equatorial plate during metaphase, choice (B), the initial connection of the microtubule to the kinetochore occurs in prophase.

A physician is attempting to diagnosis a patient's mental disorder based on a set of symptoms. The confirmed symptoms currently include appetite disturbance, substantial weight change, decreased energy, feelings of worthlessness, and excessive guilt What two disorders could these symptoms indicate? A. major depressive and bipolar disorders B. Dissociative amnesia and depersonalization/derealization disorder C. Alzheimer's disease and Parkinson's disease D. Specific phobia and panic disorder

A. The symptoms listed indicate a major depressive episode. However, depressive episodes can be a part of bipolar disorders, which also contain manic episodes. Thus, if manic episodes have not yet been asked about, one cannot choose depression or bipolar disorder as the correct diagnosis.

Consider the two sets of quantom numbers shown in the table, which describe two different electrons in the same atom n l m1 ms 2 1 1 +1/2 3 1 -1 +1/2 Which of the following terms best describes these two electrons? A. Parallel B. Opposite C. Antiparallel D. Paired

A. The terms in the answer choices refer to the magnetic spin of the two electrons. The quantom number ms represents this property as a measure of n electrons intrinsic spin. These electrons spins are parallel, in that their spins are aligned in the smae direction (Ms = +1/2 for both species)

Which of the following is LEAST likely to be required for a series of metabolic reactions? A. Tricylglycerol acting as a coenzyme B. Oxidoreductase enzyme C. Magnesium acting as cofactor D. Transferase enzymes

A. Triglycerides are unlikely to act as coenzymes for a few reasons, including their large size, neutral charge, and ubiquity in cells. Cofactors and coenzymes tend to be small in size, such as metal ions like choice (C) or small organic molecules. They can usually carry a charge by ionization, protonation, or deprotonation. Finally, they are usually in low, tightly regulated concentrations within cells. Metabolic pathways would be expected to include both oxidation-reduction reactions and movement of functional groups, thus eliminating choices (B) and (D).

A man who is color blind mates with a carrier for the trait. If the gene responsible for color blindness is found on the X chromosome, what is the chance that their son will be color blind? A. 25% B. 50% C. 75% D. 100%

B

Which of the following best describes an important property of bond energy? A. Bond energy increases with increasing bond length B. The more shared electron pairs comprising a bond, the higher the energy of that bond C. Single bonds are more difficult to break than double bonds D. Bond energy and bond length are unrelated

B

The half-life of urea is the time it takes for its concentration to fall by one-half. The graph illustrates the clearance of urea from an initial plasma concentration of 200 ng/mL. If it takes five half-lives for urea to be considered eliminated from the body, how long did it take to eliminate the urea? A.1.5 h B.2.5 h C.3.5 h D.4.0 h

B From the graph, it takes 0.5 h for the plasma concentration of urea to fall by one-half (100 ng/mL to 50 ng/mL). If it took five half-lives for urea to be considered eliminated from the body, 0.5 h should be multiplied by five. Therefore, the answer to this question is B; it took 2.5 h to eliminate urea from the body.

Some enzymes require the presence of a nonprotein molecule to behave catalytically. An enzyme devoid of the molecule is called a(n): A. holoenzyme B. apoenzyme C. coenzyme D. zymoenzyme

B. An enzyme devoid of its necessary cofactor is called an apoenzyme and is catalytically inactive.

Suppose an electron falls from n=4 to its ground state, n=1. which of the following effects is most likely to happen? A. A photon is absorbed B. A photon is emitted C. The electron moves into a p-orbital D. the electron moves into a d-orbital

B. Because the electron is moving into the n=1 shell, the only subshell available is the 1s subshell, which eliminates choices (C) and (D). There will be some energy change, however, as the electron must lose energy to return to the minimum energy ground state. That will require emitting radiation in the form of a proton.

What is the character of the bond in carbon monoxide? A. Ionic B. Polar covalent C. Nonpolar covalent D. Coordinate covalent

B. Carbon monoxide, CO, has a triple bond between carbon and oxygen, with the carbon and oxygen each retaining one lone pair. In polar covalent bonds, the difference is EN between the bonded atoms is great enough to cause electrons to move disproportionately toward the more EN atom but not great enough to transfer electrons completely. This is the case for CO. Oxygen is significantly more EN than carbon, so electrons will be disproportionately carried on the oxygen, leaving the carbon atom with a slight positive charge.

Which of the following methods would be best to separate large quantities of the following proteins? Protein pI MM PA 6.5 28,000 PB 6.3 70,000 PC 6.6 200,000 A. Ion-exchange chromatography B. Size-exclusion chromatography C. Isoelectric focusing D. Native PAGE

B. The proteins described in the question differ primarily in their molecular weights. Their pI values are very close, so ion-exchange chromatography, choice (A) is not a good choice. The question specifies a large quantity, which is better processed through chromatography than through electrophoresis - choice (C) and (D) - because the gel can only handle a small volume of protein.

Which of the following equations describes the maxumun number of electrons that can fill a subshell? A. 2l+2 B. 4l+2 C. 2l^2 D. 2l^2+2

B. This formula describes the number of electrons in terms of the azimuthal quantum number l, which ranges from 0 to n-1, with n being the principle quantum number. A table of the maximum number of electrons per subshell:

When the current in the micro-robot's circuit increases and the resistance of the circuit remains constant, the voltage of the circuit: A.decreases. B.increases. C.is constant. D.is zero.

B. This is a Physics question that falls under the content category "Electrochemistry and electrical circuits and their elements." The answer to this question is B because according to Ohm's law, current is equal to voltage divided by resistance. If current increases and resistance is constant, then voltage increases as well. It is a Scientific Reasoning and Problem Solving question because you are asked to determine and use scientific formulas to solve problems.

Knowing that the speed of light in the vitreous humor is 2.1 × 108 m/s, what is the index of refraction of the vitreous humor? (Note: The speed of light in a vacuum is 3.0 × 108 m/s.) A.0.7 B.1.4 C.2.1 D.3.0

B. This is a Physics question that falls under the content category "How light and sound interact with matter." The answer to this question is B because the index of refraction of a medium is equal to the ratio of the speed of light in vacuum to the speed of light in the medium, thus it is equal to (3.0 ´ 108 m/s)/(2.1 ´ 108 m/s) = 1.4.

If a solution containing the compounds shown in Figure 4, is injected into a gas-liquid chromatograph, the first peak observed in the gc trace is attributable to which compound? A.2-Methyl-2-butanol B.2-Methyl-2-butene C.2-Chloro-2-methylbutane D.2-Bromo-2-methylbutane

B. This is an Organic Chemistry question that falls under the content category "Separation and purification methods." The answer to this question is B because 2-methyl-2-butene will exhibit the lowest molecular weight and also the weakest intermolecular forces of attraction. This substance will therefore migrate the fastest and be the first peak in the gas chromatograph (gc) trace.

Becoming a college graduate requires hard work and diligence in academics. As such, being a college graduate could be considered a(n): a ascribed status. b achieved status. c master status. d pigeonholed status.

B. An achieved status is one that is acquired through personal efforts. This is in contract to an ascribed status, choice (A), in which the status is involuntarily given based on race, ethnicity, gender, family background, and so on. A master status, choice (C), is one that influences all aspects of an individuals life. While being a college graduate is an important aspect of day-to-day life, it does not usually pervade every part of our lives.

Which of the following is specific to an animal virus and NOT to a bacteriophage A. Infection of the host cell occurs via penetration of genome through host cell membrane. B. Uncoating of a genome from the capsid coat within the host cell cytoplasm C. Destruction of the host cell during the productive cycle D. Lysis of the cell membrane after the lytic cycle

B. Animal viruses can be endosytosed whole (uncoated) through cell membrane and ten uncoating of the genome occurs in the cytoplasm. In bacteriophages, such as the T4 phage, the capsid remains outside of the cell and the central shaft injects the genome through the membrane directly. Both bacteriohages and animal viruses must get their genome into a host cell (A is true of both) There is no immediate destruction of the host cell with the productive cycle, generally, because the virus merely buds out of the cell via exocytosis (C is wrong). Lysis of the cell membrane does not occur during the lytic cycle, but this is generally true for both animal and bacterial viruses (D is not the best answer).

Antimony is used in some anti parasitic medication specifically those targeting Leishmania Donovani What type of element is it? A. Metal B. Metalloid C. Halogen D. Nonmetal

B. Antimony (Sb) is on the right side of the periodic table, but not far right enough to be a nonmetal, choice (D). It certainly does not lie far enough to the right to fall into group VIIA (Group 17), which would classify it as a hallogen, choice (C). While sources have raely classified antimony as a metal, choice (A), it is usually classified as a metaloid, choice (B).

An EMT sees himself as a bit of a rebel, but is highly sociable and is able to keep calm in an emergency. This person would likely score in the lower range of which of the following traits? a. Psychoticism b. Neuroticism c. Extraversion d. Conscientiousness

B. As a rebel and a social person, this individual would score highly on both psychoticism and extaversion, respectively. Neuroticism is associated with high emotional arousal in stressful situations, so being able to keep calm in an emergency is a sign of low neuroticism. Conscientiousness, a trait associated with being hardworking and organizing instead of impulsive, is not described by the question stem.

Adding concentrated strong base to a solution containing an enzyme often reduces enzyme activity to zero. In addition to causing protein denaturation, which of the following is another plausible reason for the loss of enzyme activity? A. Enzyme activity, once lost, cannot be recovered B. The base can cleave peptide residues C. Adding a base catalyzes protein ploymerization D. Adding a base tends to deprotonate amino acids on the surface of proteins

B. Bases can catalyze petide bond hydrolysis. Choice (A) is incorrect: enzyme activity can be recovered in at least some cases. Choice (D) is a true statement, but fails to explain the loss of enzyme activity.

Collagen consists of three helices with carbon backbones that are tightly wrapped around one another in a "triple helix." Which of these amino acids is most likely to be found in the highest concentration in collagen? A. Proline B. Glycine C. Threonine D. Cysteine

B. Because collegen has a triple helix, the carbon backbones are very close together. Thus, steric hindrance is a potential problem. To reduce that hindrance, we need small side chains, glycine has the smallest side chains of all: a hydrogen atom.

Which of the following is/are true regarding bipolar disorders? I. They have little, if any, genetic heritability II. They are associated with increased levels of serotonin in the brain III. They all require at least one depressive episode for diagnosis. A. I only B. II only C. I and II only D. II and III only

B. Bipolar disorders have been shown to be highly heritable and are associated with increased levels of norepinephrine and serotonin in the brain. Bipolar I disorder can be diagnosed with a single manic episode and does not require a major depressive episode. Bipolar II disorder requires al least one hypomanic episode and one major depressive episode. Cyclothymic disorder contains at least one hypomanic episode and dysthymia.

A person keeps his desk extremely tidy and becomes very nervous whenever things are disorganized or out of place. In which of the following stages would a psycho-dynamic therapist say the man had become fixated? A. The oral stage B. The anal stage C. The phallic stage D. The genital stage

B. Both excessive organization and excessive sloppiness are indicative of fixation in the anal stage of psycho-sexual development - what is commonly referred to as anal-retentiveness.

Which of the following is the correct sequence of the development of a mature sperm cell? A. 1. Spermatid 2. spermatocyte 3. spermatogonium 4. (2 degree) spermatocyte 5. spermatozoan B. 1. spermatogonium 2. spermatocyte 3. (2 degree) spermatocyte 4. Spermatid 5. spermatozoan C. 1. spermatozoan 2. (1 degree) spermatocyte 3. (2 degree) spermatocyte 4. spermatogonium 5. Spermatid D. 1. spermatogonium 2. (1 degree) spermatocyte 3. (2 degree) spermatocyte 4. spermatozoan 5. Spermatid

B. Diploid cells called spermatogonia differentiate into primary spermatocytes, which undergo the first meiotic division to yield two haploid secondary spermatocytes. These undergo a second meiotic division to become immature spermatids. The spermatids then undergo a seriess of changes leading to the production of mature sperm, or spermatozoa.

In some cultures, it is considered taboo for one to show too much sadness at a funeral. In other cultures, wailing and crying loudly is expected. These cultures differ in their: a. characteristic institutions. b. display rules. c. authentic selves. d. peer groups.

B. Display rules are those that dictate cultural expectations of emotion. In some cultures, sadness is considered personal and internally in others, sadness is shared externally with the community.

Which of the following statements regarding enzymes is NOT true? A. enzymes speed up reactions by lowering the energy of activation B. enzymes increase the kinetic barrier C. enzymes are not used up during reactions D. enzymes do not effect the thermodynamics of a reaction

B. Enzymes decrease the kinetic barrier, not increase it. Enzymes lower the activation energy Enzymes do not get used up in reactions Enzymes do not affect the thermodynamics of a reaction.

Which of the following is true about the differences in genome structure between eukaryotes and prokaryotes? I. A eukaryotic chromosome is linear and a prokaryotic chromosome is circular. II. In both eukaryotes and prokaryotes, DNA is wrapped around histone proteins to form a nucleosome. III. Prokaryotic genomes contain fewer repetitive sections than eukaryotic genomes. A. I only B. I and III only C. II and III D. I, II, and III

B. Eukaryotic chromosomes are linear and wrapped around histone proteins to form nucleosomes. Prokaryotic chromosomes are circular but are not wrapped around histone proteins. Prokaryotic genomes are smaller with fewer sections of repetitive sequences.

Which of the following electronic transitions would result in the greatest gain in energy for a single hydrogen electron? A. An electron moves from n=6 to n=2 B. An electron moves from n=2 to n=6 C. An electron moves from n=3 to n=4 D. An electron moves from n=4 to n=3

B. For the electron to gain energy, it must absorb energy from photons to jump up to a higher level. There is a bigger jump between n=2 and n=6 than there is between n=3 and n=4.

Having struggled for years through an economic recession, a young professional begins to buy lottery tickets every Friday. "If I won the lottery," he reasons, "I'd finally have the life I've always wanted. All my stress would go away and I could live comfortably." His thoughts regarding winning the lottery are most representative of: A. a cardinal trait. B. fictional finalism. C. functional autonomy. D. unconditional positive regard.

B. Functional finalismis comprised of internal, idealistic beliefs about the future. The assumption that winning the lottery will solve all of his problems is representative of this form of thinking. Cardinal traits, choice (A), are the traits around which one organizes his or her entire life. Functional autonomy, choice (C), is when a behavior continues after the drive behind the behavior has ceased; for example, if this young professional continued purchasing lottery tickets after winning simply because he enjoyed them, this this behavior would have gained functional autonomy. Unconditional positive regard, choice (D), is used in some forms of humanistic therapy in which the therapist believes in the internal good of the client and does not judge the client negatively for any words or actions.

At pH 7, the charge on a glutamic acid molecule is: A. -2 B. -1 C. 0 D. +1

B. Glutamic acid is an acidic amino acid because it has an extra carboxyl group. At neutral pH, both carboxyl groups are deprotonated and this negatively charged. The amino group has a positive charge because it remains protonated at pH 7. Overall, therefore, glutamic acid has a net charge of -1, and choice (B) is correct. Notic e that you do not even need to know the pI values to solve this question: as an acidicamino acid, glutamic acid must have a pI below 7.

Political campaign ads often focus on "exposing" an opposing candidates negative characteristics. In the dramalurgical approach, one would describe this as: A. bringing the front stage self to the back stage B. bringing the back stage self to the front stage C. removing the front stage self D. removing the back stage self

B. If a candidate is 'exposed", then personal characteristics that are usually shielded from public view have been brought in front of the public. This would be pulling aspects of the back stage self to the front stage. It would not be considered removing the front stage self, choice (C), because the candidate still has a public image, even if it has been tarnished.

While on the phone, a friend says: "A good friend would let me borrow the bike." This friend is using which impression management strategy? A. Managing appearances B. Alter-casting C. Ingratiation D. Self-disclosure

B. Imposing a role on another person (in this case, :good friend") is the hallmark of alter-casting. This example is also the opposite of ingratiation, choice (C), because the implication behind the statement is that one is "bed friend" if he or she does not lend the bike; ingratiation is the use of flattery or conformity to win over someone else.

When it is time to breed, salmon travel from saltwater, in which they are hypotonic, to freshwater, in which they are hypertonic. They maintain solute balance by reversing their osmoregulatory machinery when moving between the two environments. Failure to reverse this machinery when moving to their breeding grounds would be most likely result in: A. death, as cells became too concentrated to carry would normal metabolism B. death, as cells underwent lysis due to water influx. C. improved metabolic activity, as enzyme concentrations increased D. no change, because movement from hypertonic to hypotonic medium does not present osmotic challenges.

B. In a hypotonic enviroment (freshwater) the cells will have higher osmotic pressure than the surrounding enviroments. Water would tend to flow into cells, causing them to swell and eventually burst.

A 28-year-old man comes to a clinic concerned that he has pancreatic cancer. Review of his medical records shows that this is the fourth time i the past year that the patient had appeared for medical attention. No identifiable medical problem is found. When confronted with this history, he confesses that he feels relieved after being told that all of he tests are negative, but soon becomes worried again that he has cancer. Based on the information, the most likely diagnosis for this patient would be: A. major depressive disorder B. illness anxiety disorder C. conversion disorder D. narcissistic personality disorder

B. In illness anxiety disorder, the person is preoccupied with fears that he has - or will come down with - a serious disease, and these fears continue even after medical exams and tests have returned negative results.

When dissolved in water, which of the following ions is most likely to form a complex ion with H2O A. Na^+ B. Fe^2+ C. Cl^- D. S^2-

B. Iron is a transition metal. Transition metals can often form more than one ion. Iron for example, can be Fe^2+ or Fe^3+. The transition metals, in these various oxidation states, can often form hydration complexes with water. Part of the significance of these complexes is that, when a transition metal can for a complex, its solubility within the related solvent will increase. The other ions given might dissolve readily in water, but because none of them are transition metals, they will not likely form complexes.

In a neutral solution, most amino acids exist as: A. positively charged compounds B. zwitterions C. negatively charged compounds D. hydrophobic molecules

B. Most amino acids (except the acidic and basic amino acids) have two sites for protonation: the carboxylic acid and the amine. At neutral pH, the carboxylic acid will be deprotonated (-COOO-), and the amine will remain protonated (-NH+3_. ?This dipolar ion is a zwitterion, so choice (B) is the correct answer.

A single plasmid is isolated from bacterial cells. When the plasmid is digested with a restriction enzyme and then subjected to gel electrophoresis, two bands are observed on the gel. How many restriction sites for this enzyme are present in the plasmid? A. 1 B. 2 C. 3 D. 4

B. Plasmids are circular DNA, so if the plasmid is cut in two places it will produce two fragments. The general rule when cutting circular DNA with restriction enzymes is that the number of fragments you end up with is the same as the number of restriction sites.

A protein collected through affinity chromatography displays no activity even though it is found to have a high concentration using the Bradford protein assay. What best explains this finding? A. The bradford reagent was prepared incorrectly B. The active sit is occupied by a free ligand C. The protein is bound to the column D. The protein doe not catalyze the reaction of interest

B. Protein activity and concentration are generally correlated. Because we have a high concentration of protein, we expect a high activity unless the protein has been damaged or inactivated in some way. The protein could have been inactivated by experimental conditions like detergents, heat, or pH; however, these are not answer choices. Rather, we must consider how the experimental procedure works. Protein elutes off of an affinity column by binding free ligand. In this situation, the binding may not have been reversed and thus the free ligand competes for the active site of the enzyme, lowering its activity.

Which of the following structures is found in bacterial cells? A. mitochondrion B. Ribosome C. Endoplasmic reticulum D. Nuclear membrane

B. Ribosomes are assemblies of protein and RNA, not organelles, and although there are differences between prokaryotes and eukaryotes, both forms of life have ribosommes (choice B). Bacteria are prokaryptes and therefore lack all cubcellular membrane-bound organelles, including mitochondira, the ER, and the nucleus.

Which of the following best reflects the difference between social action and social interaction? A. A social action refers to positive changes one makes in their society; social interaction refers to the route by which these changes occur. B. Social action refers tot he effects of a group on an individuals behavior; social interaction refers to the effects that multiple individuals all have on each other. C. Social action refers to changes in behavior caused by internal factors; social interaction refers to changes in behavior caused by external factors. D. Social action refers to changes in behaviors that benefit only the individual; social interaction refers to changes in behavior that benefit others

B. Social action is best described as the effect that a group ha on individual behavior, including social facilitation, de-individuation, the by standard effect, social loafing, and peer pressure. Social interaction describes how two or more individuals influence each other's behavior, including group polarization and groupthink.

What is the function of sodium dodecyl sulfate (SDS) in SDS-PAGE? A. SDS stabilizes the gel matrix, improving resolution during electrophoresis B. SDS solubilizes proteins to give them uniformity negative charges, so the separation is based purely on size. C. SDS raises the pH of the gel, separating multiunit proteins into individual subunits D. SDS solubilizes proteins to give them uniformity positive charges, so separation is based purely on pH.

B. Sodium dodecyl sulfate is a detergent and will digest proteins to form micelles with uniform negative charges. Because the protein is sequestered within the micelle, other factors such as charge of the protein and shape have minimal roles during separation. In essence, the protein micelles can be modeled as being spheres, depending only on size.

Splitting is a defense mechanism commonly seen with which personality disorder? A. Antisocial personality disorder B. Borderline personality disorder C. Histrionic personality disorder D. Narcissistic personality disorder

B. Splitting, the consideration of others as either "all good" or "all bad" is characteristic of borderline personality disorder.

In the Milgram shock experiment, many subjects were willing to give the maximal voltage shock because they were influenced by which psychological principle? A. deviance B. Obedience C. conformity D. compliance

B. The Milgram shock experiment showed that individuals would obey orders from authority figures even if they were not comfortable with the task at hand. Conformity and compliance, choices (C) and (D), also deal with changes in individual behavior, but are not based on the requests of an authority figure.

The atomic weight of hydrogen is 1.008 amu. what is the percent composition of hydrogen by isotope assuming that hydrogens only isotopes are 1H and 2D A. 92% H , 8% D B. 99.2% H , 0.8% D C. 99.92% H , 0.08% D D. 99.992% H ,0.008% D

B. The easiest way to approch this problem is to set up a system of two algebraic equations, where H and D are the presentages of H(mass = 1 amu) and D(mass = 2 amu), respectively. Youset up should look like the following system: H + D = 1 (percent H + percent D = 100%) 1 H + 2D = 1.008 (attomic weight calculation) Rearranging the first equation and substituting into the second yields (1-D) + 2D = 1.008 or D = 0.008. 0.008 is 0.8 %, so there is 0.008 % D.

Which of the following INCORRECTLY pairs a structure of the male reproductive system with a feature of the structure? A. Seminal vesicles - produce alkaline fructose containing secretions B. Epididymis - surrounded by muscles to raise and lower the testes C. Vas deferens - tube connecting the epididymis to the ejaculatory duct D. Cowper's glands - produce a fluid to clear traces of urine in the urethra

B. The epididymis is the site of sperm maturation at the posterior side of the testis. In the epidiymidymis, sperm gain mobility and are stored until ejaculation. It is the vas deferens (ductus deferens) that is surrounded by muscle that raises and lowers the testis to maintain a constant temperature suitable for sperm production, not the epididymis.

A jury member who initially feels that a strict penalty should be placed on the defendant votes for an even stricter penalty after deliberation with the other jury members. This behavior is best described by which social phenomenon? A. Social facilitation B. Group polarization C. Assimilation D. Socialization

B. The fact that individual opinions became more extreme during group discussion is explained by group polarization. The jury member initially felt that a strict penalty should be given, but this opinion became more extreme after conversation with the rest of the group.

What is the last point in the meiotic cycle in which the cell has a diploid number of chromosomes? A. During interphase B. During telophase I C. During interkinesis D. During telophase II

B. The first meiotic division (reductional division) separates homologous chromosomes to opposite poles of the cell during anaphase I. Near the end of telophase I, cytokinesis occurs, resulting in two haploid (n) daughter cells. Thus, during interkinesis and anaphase II, the daughter cells are already haploid, eliminating choices (C) and (D). The cell is diploid during interphase, choice (A), but remains diploid until the end of telophase I.

Which of the following is NOT a taste modality? A. Sweet B. Floral C. Savory D. Bitter

B. The five tastes are sweet, sour, salty, bitter, and umami. Floral would be related to smell rather than taste.

Which of the following is NOT a component of the functional attitudes theory? A. knowledge B. acceptance C. ego defense D. ego expression

B. The four functional areas of the functional attitudes theory are knowledge, adaptability, ego expression, and ego defense. Acceptance into a group may influence attitudes or opinions; however, this is not a part of the functional attitudes theory.

In the context of impression management, which of the following selves is most similar the ought self. A. The ideal self B. The tactical self C. The authentic self D. The presented self

B. The ought self is who others think we should be: the expectation imposed by others on us. This is most similar to the tactical self, which is the self er present to others when we adhere to their expectations. The presented self, choice (D), is a combination of the authentic, idea, and tactical selves.

Lithium and sodium have similar chemical properties. For example, both can form ionic bonds with chloride. Which of the following best explains this similarity? A. Both lithium and sodium ions are positively charged B. Lithium and sodium are in the same group of the periodic table C. Lithium and sodium are in the same period of the periodic table D. Both lithium and sodium have low atomic weights

B. The periodic table is organized into periods (rows) and groups (columns). Groups (columns) are particularly significant because they represent sets of elements with the same valence electron configuration, which in turn will dictate many of the chemical properties of those elements. Although choice (A) is true, the fact that both ions are positively charged does not explain the similarity in chemical properties; most metals produce positively charged ions. Choice (C) is not true because lithium and sodium are in the same group, not period. Finally, although lithium and sodium do have relatively low atomic weights, so do several other elements that do not share the same properties, eliminating choice (D).

How does the ideal temperature for a reaction change with and without an enzyme catalyst? A. The ideal temperature is generally higher with a catalyst than without B. The ideal temperature is generally lower with a catalyst than without C. The ideal temperature is characteristic of the reaction, not the enzyme D.No conclusion can be made without knowing the enzyme type.

B. The rate of reaction increases with temperature because the increase kinetic energy of the reactants, but reaches a peak temperature because the enzyme denatures with the disruption of hydrogen bonds at excessively high temperatures. In the absence of enzyme, this peak temperature is generally much hotter. Heating a reaction provide molecules with an increased chance of achieving the activation energy, by the enzyme catalyst would typically reduce activation energy. Keep in mind that thermodynamics and kinetics are not interchangeable, so we are not considering the impact of heat on the equilibrium position.

What organelle would be most closely associated with exocytosis of newly synthesized protein? A. lysosomes B. Golgi apparatus C. Peroxisomes D. Ribosomes

B. The secretory pathways passes through the ER to the Golgi, to secretory vesicles, to the exterior through exocytosis. Choices A and C are wrong because lysomes and peroxisomes are seperate destinations that proteins can be targeted to but are not in the secretory path leading to the cellular exterior. D is incorrect since ribosomes synthesize proteins, but the Golgi are more proximal to exocytosis.

Which of the following is an important property of the group of elements shaded in the periodic table below? A. These elements are the best electrical conductors in the periodic table B. These elements form divalent cations C. The second ionization energy for these elements is lower than the first ionization energy D. The atomic radii of these elements decrease as one moves down the column

B. This block represents the alkaline earth metals, which form divalent cations, or ions with a +2 charge. All of the elements in Group IIA have two electrons in their outermost s subshell. Because loss of these two electrons would leave a full octet as the outermost shell, becoming a divalent cation is a stable configuration for all of the alkaline earth metals. Although some of these elements might be great conductors, they are not as effective as the alkali metals, eliminating choice (A). Choice (C) is also incorrect because although forming a divalent cation is a stable configuration for the alkaline earth metals, the second ionization energy is still always higher than the first. Finally, choice (D) is incorrect because atomic radii increases when moving down a group of elements because the number of electron shells increases.

Which of the following elements has the highest electronegativity A. Mg B. Cl C. Zn D. I

B. This question requires knowledge of the trends of electronegativity within the periodic table. Electronegativity increases as one moves from left to right for the same reasons that effective nuclear charge increases. Electronegativity decreases as one moves down the periodic table because there are more electron shells separating the nucleus from the outermost electrons. In this question, chlorine is the furthest toward the top-right corner of the periodic table.

Which of the following molecules contains the oxygen atom with the most negative formal charge? A. H2O B. CO3^2- C. O3 D. CH2O

B. To answer this question, one must understand that contribution of resonance structures to average formal charge. In choice (B), there are there possible resonance structures. Each of the three oxygen atoms carries a formal charge of -1 in two out of the three structures. This averages to approximately -2/3 charge on each oxygen atom, which is more negative than in the other answer choices. Both water and formaldehyde, choice (A) and (D) have no formal charge on the oxygen. Ozone, choice (C), has a -1/2 on two of the three oxygen and a +1 charge on the central atom.

A physician is attempting to diagnosis a patient's mental disorder based on a set of symptoms. The confirmed symptoms currently include appetite disturbance, substantial weight change, decreased energy, s feeling of worthlessness, and excessive guilt What should the physician ask about to distinguish between the two possible disorders affecting that patient? A. Whether the patient has amnesia B. Whether the patient has also had manic episodes C. Whether the patient his irrationally afraid of anything D. Whether the patient has experienced difficulty performing familiar tasks

B. To determine if this patient has major depressive disorder or bipolar disorder, the presence of manic (or hypomanic) episodes should be confirmed. Bipolar disorders contain manic (or hypomanic) episodes, while major depressive disorders does not.

Which of the following statements is/are true of linked genes? I. They always assort together during meiosis II. They are found close together on the same piece of DNA III. They lead to greater recombination than unlinked genes A. I and II only B. II only C. II and III only D. I and III only

B. II only Statement I does not hold true 100% of the time

Certain nitrogen fixing bacteria derive their nutrition from plants. The plants, in turn, benefit from the nitrogen that the bacteria supply. Which of the following best describes the relationship between the bacteria and the plants? A. mimicry B. mutualism C. commensalism D. parasitism

B. Mutualism

The formation of alpha-D-glucopyranose from Beta-D-glucopyranose is called: A. glycosidation B. mutarotation C. enantiomerization D. racemization

B?

Although the octet rule dictates much of molecular structures, some atoms can violate the octet rule by being surrounded by more than eight electrons. Which of the following is the best explanation is the best explanation for why some atoms can exceed the octet? A. Atoms that exceed the octet already have eight electrons in their outermost electron shell B. Atoms that exceed the octet only do so when bonding with transition metals. C. Atoms that exceed the octet can do so because they have d-orbitals in which extra electrons can reside D. Some atoms can exceed the octet because they are highly EN

C

Both BF3 and NH3 have three atoms bonded to the central atom. Which of the following is the best explanation for why the geometry of these two molecules is different. A. BF3 has three bonded atoms and no lone pairs, which makes its geometry trigonal pyramidal B. NH3 is nonpolar, while BF3 is polar C. NH3 has three bonded atoms and one lone pair which makes its geometry trigonal pyramidal D. BF3 is nonpolar, while NH3 is polar

C

Which of the following is the best explanation of the phenomenon of hydrogen bonding? A. Hydrogen has a strong affinity for holding onto valence electrons B. Hydrogen can only hold two valence electrons C. Electronegative atoms disproportionately carry shared electron pairs when bonded to hydrogen D. Hydrogen bonds have ionic character

C

Which of the following types of intermolecular forces provides the most accurate explanation for why noble gases can liquefy? A. Hydrogen bonding B. Ion-dipole interactions C. Dispersion forces D. Dipole-dipole interactions

C

Consider a reaction catalyzed by enzyme A with a Km value of 5x10^-6M and Vmax of 20 mol/min At a concentration of 5x10-4 M substrate, the rate of the reaction will be: A. 10 B. 15 C. 20 D. 30 ----mmol/min

C. At a concentration of 5 x 10^-4 M, there is 100 times more substrate than present at have maximal velocity. At high values (significantly larger than the vlaue of Km) the enzyme is at or near it's Vmax, which is 20 mmol/min

What is the maximum number of electrons allowed in a single atomic energy level in terms of the principle quantum number n? A. 2n B. 2n+2 C. 2n^2 D. 2n^2+2

C. For any value of n there will be a maximum of 2n^2 electrons; that is, two per orbital. This can also be determined from the periodic table. There are only two elements (H and He) that have valence electrons in the n=1 shell. Eight elements (Li and Ne) have valence electrons in the n=2 shell. This is the only equation that matches this answer.

Ionization energy contributes to an atom's chemical reactivity. Which of the following shows an accurate ranking of ionization energies from lowest to highest. A. First ionization energy of Be < second ionization energy of Be < first ionization energy of Li B. First ionization energy of Be < first ionization energy of Li < second ionization energy of Be C. First ionization energy of Li < first ionization energy of Be < second ionization energy of Be D. First ionization energy of Li < second ionization energy of Be < first ionization energy of Be

C. Ionization energy increases from left to right, so the first ionization energy of lithium is lower than that of beryllium. Second ionization energy is always larger than first ionization energy, so beryllium's second ionization energy should be the highest value. This is because removing an additional electron from Be^+ requires one to overcome a significantly larger electrostatic force.

Which of the following is an example of a negative symptom seen in schizophrenia? A. Auditory hallucinations B. Disorganized behavior C. Disturbance of affect D. Delusions

C. Negative symptoms are the absence of normal or desired behavior, which include disterbances of affect and avolition. Positive symptoms are the addition of behavior, including hallucinations, choice (A), disorganized behavior, choice (B), and delusions, choice (D).

During what phase of the menstrual cycle does progesterone concentration peak? A. Follicular phase B. Ovulation C. Luteal phase D. Menses

C. Progesterone peaks during the luteal phase, as it supports the endometrium for potential implantation of a blastula. Progesterone levels are relatively low during follicular phase and ovulation, eliminating choices (A) and (B). Withdrawal of progesterone actually causes menses, eliminating choice (D).

Both CO3^2- and CIF3 have three atom bonded to a central atom. What is the best explanation for why CO3^2- has a trigonal planar electronic geometry, while CIF3 has a trigonal bipyramidal electronic geometry? A. CO3^2- has multiple resonance structures, while CIF3 does not. B. CO3^2- has a charge of -2, while CIF3 has no charge C. CIF3 has lone pairs on it central atom, while CO3^2- has none. D. CO3^2- has lone pairs on its central atom, while CIF3 has none.

C. The central carbon in carbonate has no lone pairs. It has three resonance structures, each of which involves a double bond between the carbon and one of the three oxygens. Having made four bonds, carbon has no further orbitals for bonding or to carry lone pairs. This makes carbonate's geometry trigonal planar. Alternatively, CIF3 also has three bonds; however, chloride still maintains two extra lone pairs. These lone pairs each inhabit one orbital, meaning that the central chloride must organize five items about itself: three bonds to flourides and two lone pairs. The best configuration for maximizing the distance between all of these groups is trigonal bopyramidal. Choices (A) and (B) are true statememtns but do not account for the difference in geometry.

Which of the following best explains the inability to measure position and momentum exactly and simultaneously according to the Heisenberg uncertainty principle? A. Imprecision in the definition of the meter and kilogram B. Limits on accuracy of existing scientific instruments C. Error in one variable is increased by attempts to measure the other D. Discrepancies between the masses of nuclei and of their component particles

C. The limitations placed by the Heisenberg uncertainty principle are caused by limitations inherent in the measuring process: if a particle is moving, it has momentum, but trying to measure that momentum necessarily creates uncertainty in the position. Even if we had an exact definition of the meter, as in choice (A), or perfect measuring devices, as in choice (B), we still wouldn't be able to measure the position and momentum simultaneously and exactly.

Which amino acids contribute most significantly to the pI of a protein? I. Lysine II. Glycine III. Arginine A. I only B. I and II only C. I and III only D. II and III only

C. The overall pI of a protein is determined by the relative number of acidic and basic amino acids. The basic amino acids: arginine, Lysine and histidine and the acidic amino acids: aspartic acid glutamic acid will therefore contribute most significantly. Glycine's side chain is a hydrogen atom, so it will have the least contribution of all the amino acids.

Which of the following will decrease the percentage ionization of 1.0 M acetic acid, CH3CO2H(aq)? A.Chlorinating the CH3 group B.Diluting the solution C.Adding concentrated HCl(aq) D.Adding a drop of basic indicator

C. This is a General Chemistry question that falls under the content category "Unique nature of water and its solutions." The answer to this question is C because HCl is a strong acid that will increase the amount of H+ in solution and thus decrease the percentage of CH3CO2H that ionizes. It is a Scientific Reasoning and Problem Solving question because you are asked to reason using a scientific principle (Le Châtelier's principle) to identify that adding a strong acid to a solution of weak acid will decrease the amount of ionization of the latter.

A person, whose eye has a lens-to-retina distance of 2.0 cm, can only clearly see objects that are closer than 1.0 m away. What is the strength S of the person's eye lens? (Note: Use the thin lens formula 1/O+1/I=S.) A.-50 D B.-10 D C.51 D D.55 D

C. This is a Physics question that falls under the content category "How light and sound interact with matter." The answer to this question is C because the strength of the eye lens is equal to the inverse of the focal length of the eye lens. Its numerical value is given by (1 m)-1+(0.02 m)-1=1 D+50 D=51 D.

What type of functional group is formed when aspartic acid reacts with another amino acid to form a peptide bond? A.An amine group B.An aldehyde group C.An amide group D.A carboxyl group

C. This is an Organic Chemistry question that falls under the content category "Structure, function, and reactivity of biologically-relevant molecules." The answer to this question is C because the functional group that forms during peptide bond formation is known as an amide group. It is a Knowledge of Scientific Concepts and Principles question since you are asked to recognize the structural relationship between free amino acids and peptides.

Researchers discover that polymorphism in the DRD2 gene can be associated with thrill-seeking behavior, and that individuals with certain forms of the gene are more likely to become extreme athletes and have more dangerous hobbies. Which of the following theories is supported by this discovery? I. The social cognitive perspective II. The behavioral perspective III. The biological perspective A. I only B. III only C. I and III only D. II and III only

C. This research supports a link between genetic expression and behavior, which is a central tenet of the biological perspective. The social cognitive perspective also holds that people's behaviors and traits shape their environments, which in tern have an effect on their identity, so the discovery also supports this perspective. Behaviorism is not supported, as the discovery is not related to rewards and punishments.

A bureaucracy is a specific example of a(n): A. immediate network B. primary group C. organization D. reference group

C. A bureaucracy is an example of an organization, specifically one with the goal of performing complex tasks as efficiently as possible. Immediate networks and primary groups, choices (A) and (B), are characterized by strong, intimate bonds, which are not commonly seen in bureaucracies. Reference groups, choice (D), are those to which we compare ourselves for various characteristics.

A child speaks in sentences of at least three words, but makes gramatical errors including misuse of the past tense. How old is this child likely to be? A. 14 months B. 22 months C. 30 months D. 5 years

C. A child who speaks in three-word sentences but has not yet mastered most of the fundamental rules of language, including part tense, is likely to be between two and three years old.

Which theory of motivation is most significantly informed by Darwin's theory of evolution? A. Arousal theory B. Drive reduction theory C. Instinct theory D. Incentive theory

C. According to Darwin's theory of evolution, all species have instincts that help them survive. The instinct theory of motivation states that people are motivated to act based on instincts that they are programmed to exhibit.

Which of the following best describes the impression management strategy of aligning actions? A. Adhering to the behaviors that are expected for a given role in society B. Relieving tension brought about by holding conflicting views in one's head C. Providing socially acceptable reasons to explain unexpected behavior D. Dictating that members of a group should follow similar practices as one another.

C. Aligning actions in an impression management technique in which one provides socially acceptable reasons for unexpected behavior. This may manifest as providing an excuse for poor performance or laughing off an inappropriate comment as a joke.. Tension created from having conflicting thoughts or opinions, as mentioned in choice (B), refers to cognitive dissonance.

metals are often used for making wires that conduct electricity. Which of the following properties of metals explains why? A. Metals are malleable B. Metals have low electronegativity C. Metals have valence electrons that can move freely D. Metals have high melting points

C. All four descriptions of metals are true, but most significant property that contributes to the ability of metals to conduct electricity is the fact that they have valence electrons that can move freely. Malleability, choice (A), is the ability to shape a material with a hammer, which does not play a role in conducting electricy. The low electronegativity and high melting points of metals choice (B) and (D), also do not play a major role in the conduction of electricity.

A particular alpha helix is known to cross the cell membrane. Which of these amino acids is most likely to be found in the transmembrane portion of the helix A. Glutamate B. Lysine C. Phenylalanine D. Aspartate

C. An amino acid likely to be found in a transmembrane portion of an alpha helix will be exposed to a hydrophobic environment, so we need an amino acid with a hydrophobic side chain. The only choice that has a hydrophobic side chain is choice (C), phenylalanine. The other choices are all polar or charged.

What determines the length of an elements atomic radius? I. The number of valence electrons II. The number of electron shells III. The number of neurons in the nucleus A. I only B. II only C. I and II only D. I, and II, and III

C. Atomic radius is determined by multiple factors. Of the choices given, the number of valence electrons does have an impact on the atomic radius. As one moves across a period (row), protons and valence electrons are added, and the electrons are more strongly attracted to the central protons. This attraction tightens the atom, shrinking the atomic radius. The number of electrons shells is also significant, as demonstrated by the trend when moving down a group (column). As more electron shells are added that separate the positively charged nucleus from the outermost electrons, the electrostatic forces weakened, and the atomic radius increases. The number of neutrons is irrelevant because it does not impact these attractive forces.

Which of the following is the most accurate, from least organized to most organized. A. Deoxyribose, nucleotide, nucleoside, DNA helix, nucleosome, chromatin B. Deoxyribose, nucleoside, nucleotide, DNA helix, chromatin, nucleosome C. Deoxyribose, nucleoside, nucleotide, DNA helix, nucleosome, chromatin D. Deoxyribose, nucleotide, nucleoside, nucleosome, DNA helix, chromatin

C. Deoxyribose, nucleoside, nucleotide, DNA helix, nucleosome, chromatin. Deoxyribose is the sugar component of the DNA backbone. A nucleoside is a deoxyribose with the nitrogenous base attached; a nucleotide is a more organized than a nucleoside and includes phosphate groups Double stranded DNA winds around histones octamers to form nucleosomes which are further packaged and condensed to form chromatin.

Which of the following is true with regard to a major depressive episode? A. It may last less than two weeks B. It must involve thoughts of suicide or a suicide attempt C. It may involve a decrease in sleep D. It must involve feelings of sadness

C. Depression is marked by a persion of at least two weeks in which the patient has five of nine cardinal symptoms, one which must be a depressed mood or lack of interest (anhedonia). Whoch decreased need for sleep is commonly seen in manic episodes, it may also appear in depression as sleep disturbance is one of the nine cardinal symptoms. Not all depressed individuals are suicidal, as in choice (C). In oldermen, depression may often manifest as anhedonia without feelings of sadness, invalidating choice (D).

Which of the following processes occur in the matrix of the mitochondria? I. electron transport chain II. glycolysis III. Krebs cycle IV. Fatty acid oxidation A. I only B. I, III, and IV C. III and IV D. III only

C. Electron transport occurs across the inner membrane of the mitochondria. Glycolysis occurs in the cytoplasm. The Krebs cycle and fatty acid oxidation both occur in the matrix of the mitochondria.

Some properties of atoms can be predicted, to some extent, by their location within the periodic table. Which property or properties increases in the direction of the arrows shown? Arrow up from the bottom (^), and from left to right (>) I. Electrronegativity II. Atomic Radius III. First ionization energy A. I only B. I and II only C. I and III only D. II and III only

C. Electronegativity describes how strong an attraction an element will have for electrons in a bond. A nucleus with a larger effective nuclear charge will have a higher electronegativity; Z eff increases toward the right side of a period. A stronger nuclear pull will also lead to increased first ionization energy, as the forces make it more difficult to remove an electron. The vertical arrow can be explained by the size of the atoms. As size decreases, the positive charge becomes more effective at attracting electrons in a chemical bond (higher electronegativity), and the energy required to remove an electron (ionization energy) increases.

Certain ovarian tumors called granulosa cell tumors are known to produce excessive levels of estrogen. A physician who diagnoses a granulosa cell tumor should look for a secondary cancer in which of the following parts of the reproductive tract? A. Fallopian tube B. Cervix C. Endometrium D. Vagina

C. Estrogen is known to cause growth of the endometrial lining during the follicular phase of the menstrual cycle, and its levels stay high during the luteal phase to promote vascularization and grandularization of the is tissue. Excessive levels of estrogen may provide a strong enough signal for cell growth to promote tumor formation, or even cancer. The other tissues listed in this question require estrogen for development, but are not strongly associated with rapid tissue growth due to estrogen.

Which of the following amino acids has a chiral carbon in it's side chain? I. Serine II. Threonine III. Isoleucine A. I only B. II only C. II and III only D. I, II, and III only

C. Every amino acidexcept glycine has a chiral alpha carbon, but only two of the 20 amino acids - threonine and isoleucine - also have a chiral carbon in their side chains as well. Thus, the correct answer is choice (C). Just as only one configuration is normally seen at the alpha carbon, only one configuration is seen in the side chain of a chiral carbon.

Hormones are found in the body in very low concentrations, but tend to have a strong effect. What type of receptor are hormones most likely to act on? I. Ligand-gated ion channels II. Enzyme-linked receptors III. G protein-coupled receptors A. I only B. III only C. II and III only D. I, II, and III

C. For a ligand present in low quantities to have a strong action, we expect it to initiate a second messenger cascade system. Second messenger systems amplify signals because enzymes can catalyze more than once while they are active, and often activate other enzymes. Both enzyme-linked receptors and G protein receptors use second messenger system, while ion channels do not.

Which of the following correctly pairs the stage of development of an egg cell with the relevant point in a woman's life cycle? A. From birth to menarche - prophase II B. At ovulation - metaphase I C. At ovulation - metaphase II D. At fertilization -prophase II

C. From the time of birth until shortly before ovulation, all egg cells are arrested at the prophase stage of meiosis I. These cells are referred to as primary oocytes. At ovulation, the egg cell has completed meiosis I and is now arrested in metaphase II as a haploid cell called a secondary oocyte. When a sperm penetrates the outer layer of the secondary oocyte, it completes meiosis II to become a mature ovum.

Which of the following is the least likely to cause denaturation of proteins? A. heating the protein to 100 degrees centigrade B. adding 8 M urea C. Moving it to a more hypotonic enviroment D. adding a detergent such as sodium dodecyl sulfate

C. High salt concentrations and detergents can denature a protein, as can hgih tempatures. But moving a protein to a hypotonic enviroment - that is, a lower solute concentration - should not lead to denaturation.

Each individual in a group of teenagers is asked to estimate the height of a tree. One individual estimates the height to be 25 feet, but after discussing with the group is convinced that the height is likely closer to 40 feet. Which type of conformity is seen here? A. Normative B. Identification C. Internalization D. Compliance

C. Internalization refers to the type of conformity in which an individual changes his or her outward opinion to match the group and also personally agrees with those ideas.

A researcher wants to clone a gene of interest into a plasmid. She plans on starting with genomic DNA, then cutting the gene of interest out. Then she plans on cutting open the plasmid and finally, ligating the genes into the plasmid. Which of the following will she need? I. restriction endonuclease II. DNA ligase III. taq polymerase A. I only B. II only C. I and II D. I, II and III

C. Item one is required. In order to cut the gene out of the genomic DNA sample, the researcher will require restriction endonuclease enzymes. The same enzymes will be required to cut open the plasmid. Item two is required. In order to ligate the gene of interest into the plasmid, the researcher will require DNA ligase. Item three is not required. Taq polymerase is used in PCR and would not required for this cloning experiment.

Which of the following viruses should be able to reproduce successfully if they carry RNA-dependent RNA polymerase into their host cell? I. (+) RNA viruses II. (-) RNA viruses III. ds DNA virus A. I only B. II only C. I and II only D. I and III only

C. Item one is true: (+) RNA viruses must at least code for RNA-dependent RNA polymerase in their genome. However, if they are carrying the enzyme, they should be able to use it to make their template strand of RNA (the (-) strand) for later genome replication. Item two is true: (-) RNA viruses must always carry at least one copy of RNA - dependent RNA polymerase in order to make strands of (+) mRNA that can be used for translation into protein. Item three is false: a ds virus would not require RNA-dependent RNA polymerase to make either RNA or replicate its genome. IT must encode enzymes for dNTP synthesis and RNA replication.

A woman advances through the ranks of a company, eventually becoming the CEO. Which of the following Jungian archetypes reflects this woman's drive to be successful within the company? A. The persona B. The anima C. The animus D. The shadow

C. Jung saw the driver for power and success as typical male traits, so Jung would say this woman is exercising her "inner man." The animus is the archetype that more closely reflects this quality.

The conversion of ATP to cyclic AMP and inorganic phosphate is most likely catalyzed by which of the following class of enzymes? A. Ligase B. Hydrolase C. Lyase D. Transferase

C. Lyases are responsible for the breakdown of a single molecule into two molecules with the addition of water or the transfer of electrons. Lyases often form cyclic compounds or double bonds in the products to accommodate this. Water was not a reactant, and no cofactor was mentioned; thus, lyase, choice (C) remains the best answer choice.

An individual who is phenotypically female is found to have only one copy of a disease-carrying recessive allele on the X chromosomes=, yet she demonstrates all of the classic symptoms of the disease. Geneticists determine she has a genotype that likely arose from nondisjunction in one of her parents. What is the likely genotype of this individual. A. 46, XX (46 chromosomes, with XX for sex chromosomes) B. 46, XY C. 45, X D. 46, XXY

C. Nondisjunction refers to the incorrect segregation of homologous chromosomes during anaphase I, or of sister chromatids during anaphse II. In either case, one daughter cell ends up with two copies of related genetic material, while the other receives zero. Immediately, this should eliminate choice (A) and (B), which show a normal complement of chromosomes (46). An individual who has only one recessive disease-carrying allele, and yet still express the disease, likely does not have a dominant allele for the given trait. This is seen in males, who are hemizygous for many X-linked genes, and can also be seen in woman with Turner syndrome (45,X), who have only one X chromosome. Thus, choice (C) is the answer.

Which of the following statements is most likely to be true of nonpolar R groups in aqueous solution? A. They are hydrophillic and found buried within proteins B. They are hydrophilic and found on protein surfaces C. They are hydrophobic and found buried within proteins D. They are hydrophobic and found on protein surfaces

C. Nonpolar groups are not capable of forming dipoles or hydrogen bonds; this makes them hydrophobic. Burying hydrophobic R groups inside of proteins means they don't have to interact with water, which is polar. This makes choice (C) correct. Choices (A) and (B) are incorrect because nonpolar molecules are hydrophobic, not hydrophilic; choice (D) is not true because they are not often found on protein surfaces.

In a personality survey, which set of twins would be expected to score the most similarly? A. Identical twins raised in different homes B. Fraternal twins raised in different homes C. Identical twins raised in the same home D. Fraternal twins raised in the same home

C. Personality is seen to be somewhat hereditary, as monozygotic, or identical twins have been seen to express more of the same personality traits. However, environment is also a factor. Thus, identical twins raised in the same home would be expected to have the most similar personalities.

In the group setting, the mentality of "If you aren't with us, you're against us" is most representative of which factors of groupthink? A. Illusion of invulnerability B. Illusion of morality C. Pressure for conformity D. Self-censorship

C. Placing a spoken or unspoken expectation on individuals to agree with the ideas of the group is best described as pressure for conformity.

Primary groups differ from secondary groups in that: A. primary groups are shorter lived than secondary groups B. Primary groups are larger than secondary groups C. primary groups are formed of stronger bonds than secondary groups D. Primary groups are assigned while secondary groups are chosen

C. Primary groups have direct and close bonds between members, providing warm, personal, and intimate relationships to its members. Secondary groups, in contrast, from superficial bonds and tend to last for a short period of time.

Each of the following responses to stress is considered maladaptive EXCEPT: A. drug use B. social withdrawal C. progressive muscle relaxation D. avoiding the stressor

C. Progressive muscle relaxation is a relaxation technique demonstrated to help reduce stress in a manner that is beneficial to the body and the psyche. The other methods described here, including avoidance of the stressor, choice (D), serve to increase stress or merely change the source of the stress.

All of the following processes occur in bacteria EXCEPT A. electron transport B. simultaneous transcription and translation C. mRNA processing D. regulation of transcription

C. Prokaryotic (bacterial) mRNA can be immediately translated, no processing is required. This is one reason why transcription and translation can proceed simultaneously Although not compartmentalized as in eukaryotes, bacteria do participate in electron transport on their cell membranes. Regulation of transcription does occur, in fact some of the best-understood transcriptional regulation mechanisms are found in bacteria.

Which of the following would NOT be required to run a polymerase chain reaction (PCR)? A. Taq DNA polymerase B. Template DNA C. RNA polymerase D. Primers

C. RNA polymerase is used during transcription, not replication. The requirements for the polymerase chain reaction are similar to what is required in a cell for DNA replication. Thus, DNA polymerase, template DNA, and primers.

Which of the following is NOT a dimension of the system for multiple level observation of groups (SYMLOG)? A. Friendliness vs. unfriendliness B. Dominance vs. submission C. Conformity vs. contrast D. Instrumentally controlled vs. emotionally expressive

C. SYMLOG is a method for analyzing group dynamics and considers groups along three dimensions: dominant vs. submissive, friendliness vs. unfriendliness, and instrumentally controlled vs. emotionally expressive.

Which of the following best describes the sociological definition of a status? A. The emotional state of a social interaction B. Expectations that are associated with a specific title in society C. A position is society used to classify an individual D. A means to describe one's peers

C. Status is a position is society used to classify a person and exists in relation to other statuses. The specific behaviors associated with this status, choice (B), best describes a role.

An alpha helix is most likely to be held together by: A. disulfide bonds B. hydrophobic effects C. hydrogen bonds D. ionic attractions between side chains

C. The alpha-helix is held together primarily by hydrogen bonds between the carboxyl groups and amino acids. Disulfide bridges, choice (A), and hydrophobic affects, choice (B), are primarily involved in tertiary structures, not secondary,. Even if they were charged, the side chains of amino acids are too far apart to participate in strong interactions in secondary structure.

A 36-year-old who works from home is referred for evaluation. He is reluctant to venture out to meet with other people and rarely has people to visit. When selected for a company-wide award, he refused to have his picture taken for the company newsletter. During an assessment,e he averts his face and asks the examiner to "stop looking at me." Although he is average in appearance, he is convinced that his face is ugly and misshapen. The most likely diagnosis for this man would be" A. schizophrenia B. obsessive-compulsive disorder C. body dysmorphic disorder D. schizoid personality disorder

C. The central issue is the negative appraisal of his own appearance, indicating body dysmorphic disorder. Thus, it is likely that all other symptoms this patient is experiencing arise from this disorder. The other disorders listed in the answer choices cannot explain all the symptoms.

Erythrobastosis fetalis is a condition in which fetal blood cell antigens elicit an immune system responce from the mother, causing clumping of the fetal blood cells. This antigenic factor is called the Rh factor, and is inherited through a dominant allele. What is the most likely status of the mother and the fetus in such a situation? A.Both the fetus and the mother are Rh+ B. The fetus is Rh- and the mother is Rh+ C. The fetus is Rh+ and the mother is Rh - D. Both the fetus and the mother are Rh -

C. The fetus's positive Rh factor is seen as foreign to the mother's immune system, which is Rh-, so it illicit an immune response.

How does the gel for isoelectric focusing differ from the gel for traditional electrophorsis? A. Isoelectric focusing uses a gel with much larger pore size to allow for complete migration B. Isoelectric focusing uses a gel with SDS added to encourage a uniform negative charge C. Isoelectric focusing uses a gel with a pH gradient that encourages a variable charge D. The gel is unchanged in isoeletric focusing; the protein mixture is treated before loading

C. The gel is isoelectric focusing uses a pH gradent. When a protein is in a region with a pH above its pI, it is negatively charged and moves toward the anode. When it is in a pH region below its pI, it is positively charged and movies toward the cathode. When the pH equals the pI, the migration of the protein is halted.

Which of the following is NOT a difference that would be allow one to distinguish a prokaryotic and a eukaryotic cell? A. Ribosomal subunit weight B. Presence of a nucleus C. Presence of a membrane on the outside surface of the cell. D. Presence of membrane-bound organelles

C. The main differences between prokaryotes and eukaryotes include: prokaryotes do not have a nucleus, while eukaryotes do, eliminating choice (B); prokaryotes have ribosomal subunits of 30S and 50S, while eukaryptes have ribosomal subunits of 40S and 60S, eliminating choice (A); and prokaryotes do not have membrane bound organelles, whereas eukaryotes do, eliminating choice (D). The presence of a membrane on the out surface of the cell could not distinguish a prokaryotic cell from a eukaryotic one because both gram-negative bacteria and animal cells share this feature.

Which of the following proteins is most likely to be found extracellularly? A. Tubulin B. Myosin C. Collagen D. Actin

C. The most prevalent extracellular proteins are keratin, elastin, and collagen. Tubulin and actin are the primary cytoskeletal proteins, while myosin is a motor protein.

A disorder of the pineal gland would most likely result in which of the following disorders? A. High blood pressure B. Diabetes C. Insomnia D. Hyperthyroidism

C. The pineal gland is responsible for producing melatonin, which controls the body's circadian rhythm. Insomnia would be a disturbance of this circadian rhythm, and may be attributable to a pineal gland disorder in some cases. Hypertension, diabetes, and hyperthyroidism would be unrelated to issues with the pineal gland.

Tay-sach's disease is a genetic disorder transmitted through an autosomal recessive gene. If a male heterozygous carrier and a female heterozygous carrier have a first child who is homozygous for the normal trait, what is the chance that the couple's second develope Tay-Sach's? A. 1/16 B. 1/8 C. 1/4 D. 1/2

C. The probability is the same regardless of the first child's genotype.

Which of the following lab techniques is described by the following steps? Step 1: Seperate DNA fragments on a gel Step 2: Transfer fragments to a nitrocellulose filter Step 3. Probe the filter for the target DNA sequence with hybridized probes A. Radioimmunoassay B. ELISA C. Southern Blotting D. Conjugation

C. The steps describe Southern Blotting. Radioimmunoassy uses radioactiviely labeeled antibodies to find target proteins or other target antibodies. ELISA uses enzymatically labeled antibodies to find target proteins or other target antibodies. Conjugation is a means by which bacteria can increase genetic diversity, it is not a lab technique.

A woman notices that her father has started to move his fingers in such a way that it looks like he is rolling something, despite nothing actually being there. She also notes slowed movement and a shuffling gait. Which neurotransmitter is likely to be present in the decreased levels in her father's brain? A. Epinephrine B. Histamine C. Dopamine D. Serotonin

C. The symptoms indicate that the woman's father likely has PArkinson's disease. This disease is caused by decreased dopamine production in the substantia nigra.

How many distinct tripeptides can be formed from one valine molecule, one alanine molecule, and one leucine molecule? A. 1 B. 3 C. 6 D. 27

C. There are three choices for the first amino acid, leaving two choices for the second, and one choice for the third. Multiplying these numbers gives us a total of 3 x 2 x 1 = 6 distinct tripeptides. (Using the one-letter codes for valine (V), alanine (A), and leucine (L), those si tripeptides are VAL, VLA, ALV, AVL, LVA, and LAV.)

Your neighbor asks you to check her mail while she is out of town and you agree. Later that day, she asks you to water her plants as well. What technique for compliance is she using in this scenario? A. lowball technique B. that's-not-all technique C. Foot-in-the-door technique D. Door-in-the-face technique

C. This is a prime example of the foot-in-the-door technique. The neighbor first asks for a small favor and, after receiving commitment, asks for a larger favor.

Which of the following does not contain tubulin? A. Cilia B. Flagella C. Microfilaments D. Centroles

C. Tubulin is the primary protein in microtubules, which are responsible for the structure and movement of cilia and flagella, eliminating choices (A) and (B). Centrioles organize microtubules in the mitotic spindle, eliminating choice (D). Mircofilaments are not composed of tubulin, but rather actin - thus answer C.

Which of the following is a form of verbal communication? A. Facial expressions B. Hand gestures C. Written text D. Body movements

C. Verbal communication uses words (whether spoken, written, or signed). Nonverbal communication uses other means of signaling emotions or ideas, such as gestures, body language, facial expressions, prosody, eye contact, and personal space.

The behavior of the individuals in the Stanford prison experiment is best explained by which of the following terms? I. Bystander affect. II. de-Individuation III. Internalization IV. Social Loafing A. I only B. III only C. II and III only D. II and IV only

C. When fulfilling particular roles, an individual's behavior can be very out of character. The changing of one's behavior (and internal ideas) to match a group is called internalization conformity. This was a key part of the experiment. The experiment also involved deindividualization, the loss of self-identity in the group setting that can lead to anti normative or violent behavior.

What is the highest-energy orbital of elements with valence electrons in the n=3 shell? A. s-orbital B. p-orbital C. d-orbital D. f-orbital

C. When n=3, l=0, 1, or 2. The highest value for l is this case is 2, which corresponds to the d subshell. Althouhg the 3d block appears to be part of the fourth period, it still has the principle quantum numer n=3. In general, the subshells within an energy shell increase in energy as follows: s<p<d<f (although there is no 3f subshell)

Upon ovulation, the oocyte is released into the: A. fallopian tube B. follicle C. abdominal cavity D. uterus

C. this subtle point about ovulation eludes most students and remains hard to believe until the organs are examined in anatomy class in medical school. The ruptured ovarian follicle releases an oocyte into the abdominal cavity, close to the entrance of the fallopian tube. With the aid of beating cilia, the oocyte is drawn into the fallopian tube, through witch it travels until it reaches the uterus. If it is fertilization does not occur, it will be expelled along with the uterine lining during menstruation.

Assuming independent assortment, how many different gametes can be produced from the genotype AaBbCc? A. 4 B.6 C.8 D. 16

C. 8 The rule is 2^n where n is the number of heterozygous genes

Females with Turner's syndrome have a high incidence of hemophilia, a recessive X-linked trait. Based on this statement, it can be inferred that females with this syndrome have: A. gained a Y B. gained a X C. lost an X D. platelets which clump together and form a plug

C. lost an X A recessive trait will only be expressed if it is present in both copies or is the only copy of the gene expressed. A normal female will need two copies to have the disease but if only one is present, the diseased allele will be expressed.

An electron returns from an excited dtate to its ground state, emitting a photon at Y = 500 nm. What would be the magnitude of the energy change if one mole of these photons were emitted? (note; h-6.626x10^-34 J x s) A. 3.98 x 10^21 J B. 3.98 x 10^19 J C. 2.39 x 10^3 J D. 2.39 x 10^5 J

While daunting at first, the problem requires the MCAT favorite equation E = hc/y, where: h=6.626x10^-34 J x s (Planck's constatat), c=3.00 x 10^8 m/s (the speed of light) and Y is the wave length of the light This question asks for the energy of one mole of photons, so we must multiply by avogadro's number, Na = 6.02 x 10^233 mol^-1 The set up is E= hc/Y x Na (6.626 x 10^43 J x s)(3.00 x 10^8 m/s) / (500x10^-9 m) x (6.02x10^23 mol^-1 =100/500 x 10^6 J = 2 x 10^5 J

All of the following are true of epimers EXCEPT: A. they differ in configuration about only one carbon B. they usually have slightly different chemical and physical properties C. they are diastereomers (with the exception of glyceraldehyde) D. they have equal but opposite optical activities

D

Despite the fact that both C2H2 and HCN contain triple bonds, the length of these triple bonds are not equal. Which of the following is the best explanation for this finding? A. In C2H2 the bond is shorter because it is between atoms of the same element. B. The two molecules have different resonance structures C. Carbon is more electronegative than hydrogen D. Nitrogen is more electronegative than carbon

D

If erythrocytes are placed into a hypertonic solution, they will: A.hemolyze B.remain the same C.swell up D. shrivel

D In a hypertonic solution, the extracellular envirmoent has a higher solute concentration. Osmotic pressure would draw water out of the cell, amking solutes more concentrated inside the cell, and making the cell shrivel.

As a gender identity, androgyny is defined as: A. low femininity, low masculinity B. high femininity, low masculinity C. low femininity, high masculinity D. high femininity, high masculinity

D. Androgyny is defined as scoring highly on scales of both femininity and masculinity. Achieving a low score on both scales, choice (A), would be considered undifferentiated, while choice (B) and (C) would be described as feminine and masculine, respectively.

A young woman of unknown age is brought by the Philadelphia police to the local emergency department for evaluation after they found her wandering in a park. She carries no purse or identification. She is unable to state her name or any details about her life, except that the name Phoenix seems familiar. The police in Arizona are contacted and find a missing persons report matching the patient's description. Based on this information, the most likely diagnosis for this patient is: A. deperonalization / derealization disorder B. dissociative identity disorder C. somatic symptom disorder D. dissociative amnesia with dissociative fague

D. Dissociative fugue is characterized by sudden travel or change in normal day-to-day activities, and occurs in some cases of dissociative amnesia. Symptoms include an inability to recall one's past or confusion about one's identity.

A mother notices that her teenage son seems to have a phobia of snakes. In the past week , on several occasions, the teenager has had more severe fear symptoms than usual, without seeing or even thinking about a snake. Which mental disorder would cause this reaction? A. Schizophrenia B. Antisocial personality disorder C. obsessive-compulsive disorder D. panic disorder

D. Exhibiting signs of panic and irrational fear without any instigating object present indicates panic disorder. The teenager may have diagnosis of both specific phobia (for snakes) and panic disorder.

After sitting in a lecture, determining that a professor is a bad teacher based on his unprofessional attire and monotone speech is an example of which type of processing? A. Knowledge route processing B. adaptive route processing C. central route processing D. peripheral route processing

D. Peripheral route processing deals with processing information that is not based on content, but instead on superficial parameters such as boring speech patterns or appearance of the speaker. Central route processing choice (C), is the processing of information through analysis of its content.

Which of the following atoms only has paired electrons in its ground state? A. Sodium B. Iron C. Cobalt D. Helium

D. The only answer choice without unpaired electrons in its ground state is helium. Recall from the chapter that a diamagnetic substance is identified by the lack of unpaired electrons in its subshell. A substance without unpaired electrons, like helium, cannot be magnetized by an external magnetic field and is actually slightly repelled. Elements that come at the end of a block (Group IIA, the group containing Zn, and the noble gases, most notably) have only paired electrons.

Which statement correctly describes how enzymes affect chemical reactions? Stabilization of: A.the substrate changes the free energy of the reaction. B.the transition state changes the free energy of the reaction. C.the substrate changes the activation energy of the reaction. D.the transition state changes the activation energy of the reaction.

D. This is a Biochemistry question that falls under the content category "Principles of chemical thermodynamics and kinetics." The answer is D because the stabilization of the transition state, not the substrate, provides binding energy that is used to lower the activation energy.

The intensity of the radiation emitted by the oxygen sensor is directly proportional to the: A.propagation speed of the radiation. B.wavelength of the radiation. C.polarization of photons emitted. D.number of photons emitted.

D. This is a Physics question that falls under the content category "How light and sound interact with matter." The answer to this question is D because the energy of electromagnetic radiation is directly proportional to the number of photons, and the intensity of electromagnetic radiation is defined as energy emitted per unit time. Thus, intensity is directly proportional to the number of photons emitted.

What is the energy of the photons emitted by the LED at a frequency of 610 THz? (Note: h = 6.6 × 10-34 J·s) A.9.2 × 10-12 J B.1.6 × 10-16 J C.1.1 × 10-18 J D.4.0 × 10-19 J

D. This is a Physics question that falls under the content category "How light and sound interact with matter." The answer to this question is D because the energy of a photon of frequency 610 THz is equal to 6.6 ´ 10-34 J•s ´ 610 ´ 1012 Hz = 4 ´ 10-19 J.

What is the work generated by a healthy adult who circulates 9 L of blood through the brachial artery in 10 min? A.2 kJ B.12 kJ C.20 kJ D.120 kJ

D. This is a Physics question that falls under the content category "Translational motion, forces, work, energy, and equilibrium in living systems." The answer to this question is D because a flow of 9 liters in 10 minutes means a flow rate of 900 mL/min, and according to the graph, it corresponds to a power of 200 W. The work is then 200 W ´ 600 s = 120 kJ.

Which of the following statements does NOT correctly describe the dehydration of malic acid to fumaric acid and maleic acid? A.The reaction occurs most readily with tertiary alcohols. B.The reaction involves the loss of a water molecule. C.The reaction has a carbocation intermediate. D.The reaction is stereospecific.

D. This is an Organic Chemistry question that falls under the content category "Nature of molecules and intermolecular interactions." The answer to this question is D because the fact that both fumaric and maleic acid are produced means that the dehydration of malic acid is NOT stereospecific. It is a Data-based and Statistical Reasoning question because you are asked to analyze and interpret data (the outcome of a particular organic chemical reaction) in order to draw a conclusion.

Acetic acid and ethanol react to form an ester product as shown below. In determining which reactant loses the -OH group, which of the following isotopic substitutions would be most useful? A.Replace the acidic H of acetic acid with D. B.Replace the alcoholic H of ethanol with D. C.Replace the carbonyl oxygen of acetic acid with O-18. D.Replace the hydroxyl oxygen of ethanol with O-18.

D. This is an Organic Chemistry question that falls under the content category "Structure, function, and reactivity of biologically-relevant molecules." The answer to this question is D because this experiment involves labeling a group which does not exchange with other groups present prior to reaction and will therefore give information about the true identity of the groups, which are exchanged during the reaction.

An 18 year-old male is completing his final moths of high school and begins to wake up early each day to run five miles in preparation for joining the army. What type of socialization is this young man experiencing? A. normative socialization B. informative socialization C. re-socialization D. anticipatory socialization

D. This young man is preparing for a life in the army, a new social setting that he will be joining. The process of preparing for future changes in environment is considered anticipatory socialization.

A researcher is studying a certain protein of interest, SAL3, which has been isolated from two strains of bacteria, Strain E and strain J. The researcher identifies Sal3 via a western blot and discovers that SAL3 from strain E is the expected molecular weight but SAL3 from strain J is shorter. Amino acid sequencing conforms that the SAl3 from stain J contains fewer amino acids. Which of the following best explains this result? A. Stain E underwent a conservative missense mutation in the open reading frame coding for SAL3. B. Strain E was allowed to grow for a longer time in culture than strain J C. Strain J could have undergone a nonsence mutation only, in the opening reading frame coding for Sal3. D. Strain J could have undergone either a nonsense or frameshift mutation in the open reading frame codoing for Sal3.

D. A conservative mutation leads to a change in one amino acid, to another with similar chemical characteristics (for example, the codon for leucine mutated to the condon for isoleucine). This would not explain they the protein in strain J is shorter. Different growth times also does not explain why the strains have different length Sal3 proteins Both choice C and D say that a stop codon too early in the the opening reading frame would lead to translation of a shorter protein. However, choice D is better than choice C because it allows for the possibility of a frame shift mutation. A frame shoft mutation is the insertion or deletion of nucleotides leading to a change in the reading frame of the transcript. This can lead to the creation of a stop codon when there should not be one. For example, if the normal reading frame of Sal3 is UUU CUA GGG this would be code for phe-leu-gly. But, if the first U is deleted, UUC UAG GG results and this would code for phe-stop. Therefore, frame shift mutations can also lead to a shorter protein.

In Kartagener's syndrome, defective dynein is produced causing a paralysis of microtubule-based movement of flagellae and cillia. One would expect to find all of the following outcomes EXCEPT: A. male fertility B. ectopic pregancy in women C. chronic lung infections D. failure to ovulate in women

D. A male with Kartegener's syndrome would be infernal due to immobile sperm, eliminating A. In females, ova would not enter the Fallopian tubes normally, due to the lack of cilia, causing increased risk of ectopic pregnancy, eliminating B. The luings also require cilia to remove bacteria and other particulates, eliminating C. Ovulation, however, is determined by level of circulating hormones and will not be affected by lack of dynein.

Each of the following is true EXCEPT: A. PCR cycles through 3 tempatures: high temp for template denaturing, low temp for primer annealing and medium temp for polymerization B. proteins are probed with antibodies in a western blot C. During gel elctrophoresis, negative DNA movies to the positive end of the gel in a size dependent manner D. DNA is probed with DNA or RNA in a northern blot.

D. A northern blot is when RNA is probed with either DNA or RNA. Choice D - incorrect) Choice A accurately describes the temperature changes during PCR. (A) Antibodies are used to ID proteins in a western blot. (choice B) When DNA is separated on a gel, the negative charge of the phosphate backbone causes DNA to migrate towards the positive end of the gel. Small fragments of DNA move quickly and large fragments more slowly. (choice C)

A medical student is feeling a high level of stress due to upcoming exams and pressure from his family to engage in activities at home. He chooses to go the gym for a workout to help himself relax. This workout is which type of stress? A. Hassle B. Frustration C. Distress D. Eustress

D. A positive stressor creates eustress. Because working out is used to relax, it is considered a eustress. Hassle, choice (A), and frustration, choice (B), are both types of distress, choice(C), or negative stressors.

Colchicine is a compound which interferes with the formation of microtubles. Which of the following would be affected LEAST by the administration of colchicine? A. Mitotic spindles B. Flagellae C. Organelle movement D. Amoeboid motility of cells

D. Actin microfilaments, not microtubules, are responsible for amoeboid movement of cells. The other choices are wrong since microtubules are a key component of mitotic spindles, flagella, and the scaffold that organelles interact with to move within the cytoplasm.

In addition to being a freestanding diagnosis, agoraphobia is most often seen in association with which other psychiatric diagnosis? A. obsessive-compulsive disorder B. avoidance personality disorder C. generalized anxiety disorder D. panic disorder

D. Agoraphobia, or fear of places or situations in which would be difficult to escape, is commonly seen in panic disorder. Concern about having a panic attack in public may make these individuals fearful of leaving their home.

Which of the following is NOT a component of all trimeric G proteins? A. G alpha B. G beta C. G Y D. G i

D. All trimeric G proteins have alpha, beta, and Y subunits - choices (A), (B), and (C), respectively. Gs, Gi, and Gq are subtypes of the alpa G subunit of the trimeric G protein and differ depending on the G protein-coupled receptor's function.

In lysine, the pKa of the side chain is about 10.5. Assuming that the pKa of the carboxyl and amino groups are 2 and 9 respectively, the pI of lysine is closest to: A) 5.5 B) 6.2 C) 7.4 D) 9.8

D. Because lysine has a basic side chain, we ignore the pKa of the carboxyl group, and average the pKa of the side chain and the amino group; the average of 9 and 10.5 is 9.75, which is the closest to choice (D).

A-enzyme1->B-enzyme 2->C In the equation above, substrate C is allosteric inhibitor to enzyme 1. Which of the following is another mechanism necessarily caused by substrate C? A. Competitive inhibition B. Irreversible inhibition C.feedback enhancement D. negative feed back

D. By limiting the activity of enzyme 1, the rest of the pathway is slowed, which is the definition of negative feedback. Choice (A) is incorrect because there is no competition for the active site with allosteric interactions. While many products do indeed competitively inhibit an enzyme in the pathway that creates them, this is an example of an allosterically inhibited enzyme. There is not enough information for choice (B) to be correct because we aren't told whether the inhibition is reversible. In general, allosteric interactions are temporary. Choice (C) is incorrect because it s the opposite of what occurs when enzyme 1 activity is reduced.

Which of the following is a reason for conjugating proteins? I. To direct their delivery to a particular organelle II. To direct their delivery to the cell membrane III. To add a cofactor needed for their activity A. I only B. II only C. II and III only D. I, II, and III

D. Conjugated proteins can ahve a lipid or charbohydrate "tags" added to them. These tags can indicate that these proteins should be directed to the cell membrane (especially lipid tags) or to specific organelles (such as the lysosome). They can also provide the activity of the protein; for example, the heme group in hemoglobin is needed for it to bind to oxygen. Thus, choice (D) is the correct answer.

A certain cooperative enzyme has four subunits, two of which are bound to substrate. Which of the following statements can be made? A. The affinity of the enzyme for the substrate has just increased B. The affinity of the enzyme for the substrate has decreased C. The affinity of the enzyme for the substrate is at average for this enzyme class. D. The affinity of the enzyme for the substrate is greater than with one substrate bound.

D. Cooperative enzymes demonstrate a change in affinity for the substrate depending on how many substrate molecules are bond and whether the last change was accomplished because a substrate molecule was bond or left the active site of the enzyme. Because we cannot determine whether the most recent reaction was binding or dissociation, choice (A) and (B) are eliminated. We can make absolute comparisons though. The unbound enzyme has the lowest affinity for substrate, and the enzyme with all but one subunit bound has the highest. The increae in affinity is not linear, and therefore choice (C) is not necessarily true. An enzyme with two subunits occupied must have a higher affinity for the substrate than the same enzyme with only one subunit occupied, thus, choice (D) is correct.

Which of the following would NOT be seen during pregnancy? A. High levels of hCG in the first trimester B. High levels of progesterone throughout the pregnancy C. Low levels of FSH in the first trimester D. High levels of GnRH throughout the pregnancy

D. During the first trimester of p pregnancy, the corpus luteum is preserved by human chorionic gonadotropin (hCG); hence, progesterone secretion by the corpus luteum is maintained during the first trimester. This eliminates choice (A). '''''''''''''''''''''''''''''during the second trimester, hCG levels decline, but progesterone levels rise because the hormone is now secreted by the placenta itself, eliminating choice (B). High levels of progesterone and estrogen inhibit GnRH secretion, thus preventing FSH and LH secretion and the onset of a new menstrual cycle. This eliminates choice (C) and validates choice (D).

Which of the following is NOT a method by which enzymes decrease the activation energy for biological reactions? A. Modifying the local charge environment B. Forming transient covalent bonds C. Acting as electron donors or receptors D. Breaking the bonds in the enzyme irreversibly to provide energy.

D. Enzymes are not altered by the process of catalysis. A molecule that breaks intramolecular bonds to provide activation energy would not be able to be reused.

Consider a biochemical reaction A -> B, which is catalyzed by A-B dehydrogenase. Which of the following statements is true? A. The reaction will proceed until the enzyme concentration decreases. B. The reaction will be most favorable at 0 degrees C. C. A component of the enzyme is transferred from A to B. D. The free energy change (change in G) of the catalyzed reaction is the same for the uncatalyzed reaction.

D. Enzymes catalyze reactions by lowering their activation energy, and are not changed or consumed during the course of teh reaction. While the activation energy is lowered, the free energy of the reaction (change in G) remains unchanged in the presence of an enzyme. A reaction will continue to occur in the presence or absence of an enzyme; it simply runs slower without the enzyme, eliminating choice (A). Most physiological reactions are optimal at body temperature 37 degrees C, eliminating choice (B). Finally, dehydrogenases catalyze oxidation-reduction reactions, not transfer reactions, eliminating choice (C).

Which of the following is NOT involved in cell migration? A. Dynein B. Flagella C. Actin D. Centrioles

D. From the given choices, all of them are involved in cell movement with the exception of choice (C), but are involved in mitosis, not cell migration.

The ratio of guanine-cytosine (G—C) pairs to adenine-thymine (A—T) pairs is useful in laboratory manipulation of double-stranded DNA. If a segment of DNA has a low G—C : A—T ratio, it would be reasonable to assume that this segment would: A. contain more guanine than cytosine B. contain more adenine than thymine C. require more energy to seperate the two DNA strands than would a comparable segment of DNA have a high G-C : A-T ratio D. requires less energy to seperate the two DNA strands than would a comparable segment of DNA having a high G-C:A-T ratio

D. G -C base pairs are linked by three hydrogen bonds in the double helix, while A-T base pairs are joined by only two hydrogen bonds. It takes more energy to seperate G-C base pairs, and the less G-C rich a piece of double stranded DNA is, the less energy that is required to seperate the two strands of the double helix. Note that choices A and B can be eliminated since double stranded DNA, there must be equal quantities of G and C (and A and T).

Which of the following is NOT characteristic of a bureaucracy? A. Rigidly defined work procedures B. Requirement for officials to hold an advanced degree C. Regular salary increases D. Election by constituents

D. Generally, bureaucracies are marked by six characteristics: 1. paid officials on a fixed salary 2. non-elected officials provided rights and privileges as a result of making their career out of holding office 3. regular salary increases 4. Seniority rights 5. promotions upon passing exams or milestones choice (C), officials who enter the organization by holding advanced degree or training, choice (B), responsibilities, obligations, privileges, and work procedures rigidly defined by the organization, choice (A), and responsibly for meeting the the obligations of the office one holds.

At what pH can protein A best be obtained through electrophoresis? Protein pI MM PA 4.5 25,000 PB 6.0 10,000 PC 9.5 12,000 A. 2.5 B. 3.5 C. 4.5 D. 5.5

D. In most electrophoresis experiments, we attempt to separate out one component from the others. Because we are attempting to isolate protein A only, a pH that causes protein A to be negative while protein B and C are neutral or positive will be best. pH 5.5 accomplishes this goal; proteins B and C will be positively charged. A pH of 4.5, choice (C), would make protein A neutral, and it would thus not migrate across the gel. Any neutral impurities would also remain in the well with protein A, making this pH not the best choice.

Which of the following is true regarding prophase? A. The chromosomes separate and move to opposite poles of the cell B. The spindle apparatus disappears C. The chromosomes uncoil D. The nucleoli disappear

D. In prophase, the chromatin condenses into chromosomes, the spindle apparatus forms, and the nucleoli and nuclear membrane disaooear. Choice (A) describes anaphase, whereas choice (b) and (C) describe telophase.

Which of the following is an example of intraspecific animal communication? A. A dog who barks when a stranger enters the house B. An anglerfish that uses bio-luminescent appendage to attract prey C. Bats using echolocation to detect the surrounding environment D. A cat who uses scent glands to mark his territory for other cats.

D. Intraspecific communication refers to communication between members of the same species. Interspecific communication, on the other hand, refers to communication between members of different species. Echolocation is not an example of intraspecific communication because the sender of the signal and the recipient are the same organism; this would be considered autocommunication.

Why do halogens often form ionic bonds with alkaline earth metals? A. The alkaline earth metals have much higher electron affinities than halogens B. By sharing electrons equally, the alkaline earth metals and halogens form full octets C. Within the same row, the halogens have smaller atomic radii than the alkaline earth metals D. The halogens have much higher electron affinities than the alkaline earth metals.

D. Ionic bonds are formed through unequal sharing of electrons.. These bonds typically occur because the electron affinities of the two bonded atoms differ greatly. For example, the halogens have high electron affinities because adding a single electron to their valence shells would create full valence shells. In contrast, the the alkaline earth metals have very low electron affinities and are more likely to be electron donors because the loss of two electrons would leave them with fill valence shells. Choice (A) states the opposite and is incorrect because the halogens have high electron affinity and the alkaline earth metals have low electron affinity. Choice (B) is incorrect because equal sharing of electrons is a classic description of covalent bonding, not ionic. Choice (C) is a true statement, but not relevant to why ionic bonds form.

Which of the following is most likely to be found bound to a protein in the body? A. Sodium B. Potassium C. Chloride D. Calcium

D. Ions that are not readily accessible in the cytoplasm or extracellular space are likely to be bound to a binding protein. Classically, calcium and magnesium are protein bound. Without this background knowledge, the question can still be answered. Sodium, choice (A), and potassium, choice (B), must exist in their free states to participate in action potentials. Chloride, choice (C), is readily excreted by the kidneys, which would not be true if it were protein bound. Calcium must be sequestered in both the bloodstream and intracellularly because calcium is used for muscle contraction, exocytosis (of neurotransmitters and other signals), and many other cellular processes that must be tightly regulated.

Which of the these statements concerning peptide bonds is FALSE? A. Their formation involves a reaction between an amino group and a carboxyl group B. They are the primary bonds that hold amino acids together C. They have partial double bond character D. Their formation involves hydration reactions

D. Peptide bonds are the primary covalent bond between the amino acids that make up proteins, making choice (B) incorrect. They involve a condensation reaction between the amino group of one amino acid and the carboxyl group of an adjacent amino acid, eliminating choice (A). The petide bond has a partial double bond character because the double bond can resonate between C=O and C=N. Thus, the petide bond has a partial double bond character and exhibits limited rotation, eliminating choice (C). By process of elimination, choice (D) is false: formation of the peptide bond is a condensation reaction - specifically a dehydration reaction involving the loss of water - not a hydration reaction involving the addition of water.

All of the following are well recognized multicellular eukaryotic kingdoms except: A. Animalia B. Plantae C. Fungi D. Protists

D. Protists are primarily single celled eukaryotes, although a few multicellular varieties exist (e.g. seaweeds). Animalia, Plantae, and Fungi all represent kingdoms of multicellular eukaryotes. (Note that Kingdom Fungi does include one unicellular member - yeast - but is primarily made up of multicellular organisms).

Due to electron transport in a eukaryotic cell: A. the pH in the intermembrane space is lower because protons are pumped into the matrix of the mitochondria. B. the pH in the intermembrane space is higher because protons are pumped into the matrix of the mitochondria. C. the pH in the matrix of the mitochondria is lower because protons are pumped across the inner membrane of the mitochondria D. the pH in the matrix of the mitochondria is higher because protons are pumped across the inner membrane of the mitochondria.

D. Protons are pumpoed across the inner membrane of the mitochondria, out of the matrix and into the intermembrane space resulting in an increase of the pH of the matrix

Which of the following is the equation for recombination frequency (RF)? A. RF = number of parental phenotypes / total number of parental genotypes B. RF = number of recombination phenotypes / total number of recombination genotypes C. RF = number of non-parental genotypes / total number of parental genotypes D. RF = number of recombination phenotypes / total number of progeny

D. Recombinants include those with non-parental phenotypes Works easiest when the other parent is a tester (recessive for both genes)

A researcher is running a PCR experiment to amplify her DNA of interest. Assuming she starts with a single DNA molecule, after 30 rounds of PCR, how many DNA molecules would she have? A. 2 x 30 B. 1 + 30^2 C. 30^2 D. 2^30

D. Starting with a single DNA molecule, after 1 round of PCR, the researcher would have 2 molecules of DNA, or 2^1 molecules. After 2 rounds, 4 molecules or 2^2 Following this pattern, the number of DNA molecules at the end of PCR is equal to n^n, where n= the number of rounds of PCR.

Which of the following atoms or ions has the largest effective nuclear charge? A. Cl B. Cl^- C. K D. K^+

D. The effective nuclear charge refers to the strength within which protons in the nucleus can pull on electrons. This phenomenon helps to explain electron affinity, electronegativity, and ionization energy. In choice (A), the non ionized chlorine atom, the nuclear charge is balanced by the surrounding electrons: 17 p^+/17 e^- The chlorine ion, choice (B) has a lower effective nuclear charge because there are more electrons than protons: 17 p^+ / 18 e^-. Next elemental potassium, choice (C) has the lowest effective nuclear charge because it contains additional inner shells that shield its valence electron from the nucleus. Choice (D) ionic potassium, has a higher effective nuclear charge than any of the other options do because it has the same electron configuration as Cl^- (and the same amount of shielding from inner shell electrons as neutral Cl) but contains two extra protons in its nucleus: 19 p^+ / 18 e^-

Which of he following statements correctly identifies a key difference between mitosis and meiosis? A. In metaphase of mitosis, replicated chromosomes line up in a single file; in metaphase II of meiosis, replicated chromosomes line up on opposite sides of the metaphase plate. B. During anaphase of mitosis, homologous chromosoes separate; during anaphase of meiosis I, sister chromatids separate C. At the end of telophase of mitosis, the daughter cells are identical to each other; at the end of meiosis I, the daughter cells are identical to the parent cell. D. During metaphase of mitosis, centromeres are present directly on the metaphase plate; during metaphase of meiosis I, there are no centomeres on the metaphase plate.

D. The key differences between mitosis and meiosis primarily appear during meiosis I. Of note, synapses and crossing over occur during prophase I, and homologous chromosomes are separated during meiosis I (rather than sister chromatids, as in mitosis). While the location of the centromeres relative to the metaphase plate may seem a trivial point, it is representative of the fact that homologous chromosomes line up on opposite sides of the equatorial plate in meiosis, in contrast to the alignment of each chromosome directly upon the metaphase plate in mitosis.

Which of the following correctly ranks the compounds below by ascending boiling point? I. acetone II. KCL III. Kr IV. Isopropyl alcohol A. I < II < IV < III B. III < IV < I < II C. II < IV < I < III D. III < I < IV < II

D. The key to answering this question is to understand that types of intermolecular forces that exist in each of these molecules because larger intermolecular forces correspond to higher melting points. Kr is a noble gas with a full octet, so the only intermolecular forces present are london dispersion forces. Acetone and isopropyl alcohol are both polar, so both have dipole-dipole interactions, which are stronger than dispersion forces. However, isopropyl alcohol can also form hydrogen bonds, increasing its boiling point. Finally, the strongest interactions are ionic bonds, which exist in potassium chloride.

Which of the following is NOT likely to contribute to genetic variability? A. Random fertilization of an egg by a sperm B. Random segregation of homologous chromosomes C. Crossing over between homologoud chromosomes during meiosis D. Replication of the DNA during S phase

D. The safest way to answer this question correctly is to go through each answer choice and eliminate the ones that contribute to genetic variability. The random fertilization of an egg by a sperm, the random segregation of homoloogous chromosomes during anaphase I, and crossing over between homologous chromosomes during prophase I all contribute to genetic variability during sexual reproduction because they result in novel combinations of genetic material, eliminating choices (A), (B), and (C). S stage, choice (D), should specifically not cause increased genetic variability; the DNA should be copied precisely, without error, meaning that both strands of DNA should be identical.

Cancer cells are cells in which mitosis occurs continuously without regard to quantity of the cells produced. For this reason, most chemo therapies attack rapidly dividing cells. At which point(s) in the cell cycle could chemotherapy effectively prevent cancel cell division? I. S stage II. Propahse III. Metaphase A. I only B. I and II only C. II and III only D. I, II and III

D. This question is asking us to determine at which points in the cell cycle we can prevent or at least lower the number of cells undergoing mitosis. One idea would be to prevent DNA synthesis during the S stage of the cell cycle. Without the DNA being replicated, two viable daughter cells could not be formed. Another idea would be preventing the mitotic cycle from occuring altogether in prophase by preventing spindle apparatus formation, preventing the nuclear membrane from dissolving, or other processes during this phase. Similarly, a treatment that would act on cells in the metaphase stage of the cell cycle would also interfere with the mitotic cycle. Therefore, any of the three solutions presented would be a viable option.

How many valence electrons electrons are present in elements in the thirds period? A. 2 B. 3 C. The number decreases as the atomic number increases D. The number increases as the atomic number increases

D. This question is simple if one recalls that periods refer to the rows in the periodic table, while groups or families refer to the columns. Within the same period, an additional valence electron is added with each step toward the right side of the table.

A researcher wises to incorporate a radiolabeled deoxyadenine into the genome of one of the two daughter cells that would arise as a result of mitosis. What is the latest stage of cellular development during which the radiolabeled deoxyadenine could be added to achieve this result? A. G1 B. G2 C. M D. S

D. To ensure that the labeled deoxyadenine will be incorporated into the DNA of one of the daughter cells, we have to insert the nucleotide before DNA replication has been completed. Because replication occurs during S phase, we would introduce the deoxyadenine during G1 or S stage. Because G1 proceeds S, the latest point the deoxyadenine could be added is the S stage.

During groupthink, members of the group do all of the following EXCEPT: A. stereotype members outside of the group. B. withhold opposing views. C. ignore warnings against the ideas of the group. D. create a sense of negativity against risk-taking.

D. With groupthink, a member would perform all of the actions described by the answer choices except create a sense of negativity against risk-taking; in fact, there is optimism and encouragement toward risk-taking in groupthink.

If a man with a mutant copy of Ace2 has a child with a woman that is heterozygous for the mutant Ace2 allele, what is the probability that the child will be a female and homozygous for the mutant Ace2 allele? A.0 B.0.25 C.0.75 D.1

If a man with a mutant copy of Ace2 has a child with a woman that is heterozygous for the mutant Ace2 allele, what is the probability that the child will be a female and homozygous for the mutant Ace2 allele? A.0 B.0.25 C.0.75 D.1

Which of the following isotopes of carbon is LEAST likely to be found in nature? A. ^6C B. ^12C C. ^13C D. ^14C

Recall that the superscript refers to the mass number of an atom, which is equal to the number of protons plus the number of neutrons present in an element. Sometimes a text will list the atomic number, Z, as a subscript under mass number, A. According to the periodic table, carbon contains six protons, therefore, its atomic number is 6. Isotopes all have the same number of protons, but differ in the number of neutrons. Almost all atoms with Z > 1 have at least one neutrons. Carbon is most likely to have a mass number of 12, for six protons and six neutrons, as in choice (B). Choices (C) and (D) are possible isotopes that would have more neutrons than ^12C. The ^6 isotopes is unlikely. It would mean that there are 6 protons and 0 neutrons. This would be a highly unstable isotope.

ornithine + H2O → H2NCH2CH2CH2CH2NH2 + HCO3- Which of the following terms best describes the role of ornithine decarboxylase in the reaction shown in Equation 1? A.Catalyst B.Cofactor C.Substrate D.Activator

The answer is A because ornithine decarboxylase is an enzyme and enzymes are catalysts.

Which amino acid, in addition to tryptophan, is most likely to be transported by AT1? A.Phenylalanine B.Lysine C.Arginine D.Glutamate

The answer is A because phenylalanine is a neutral amino acid just like tryptophan and shares an aromatic structure. This means it is most likely to be recognized by the same transport machinery.

In which direction does the Na+K+ ATPase transport ions across the cell membrane upon ATP hydrolysis? A.Na+ is transported out of the cell; K+ is transported into the cell. B.Na+ is transported into the cell; K+ is transported out of the cell. C.Both Na+ and K+ are transported into the cell. D.Both Na+ and K+ are transported out of the cell.

The answer is A because the Na+K+ ATPase transports 3 sodium ions out of the cell and 2 potassium ions into the cell with each ATP hydrolyzed.

The process of culturing bacteria often involves inoculation of cells on a noncellular, agar-based medium. Such a methodology would NOT result in growth of animal viruses because animal viruses: A.are obligate parasites. B.lack DNA. C.assimilate carbon. D.require essential vitamin supplements for growth.

The answer is A because viruses can only reproduce in a host cell, and are therefore obligate intracellular parasites.

Which type of bond is formed by glycogen synthase upon release of UDP? A.α-1,4-Glycosidic bond B.α-1,6-Glycosidic bond C.β-1,4-Glycosidic bond D.β-1,6-Glycosidic bond

The answer is A because the bond that is formed by glycogen synthase is the main chain linkage of glycogen, which is an α-1,4-glycosidic bond. UDP release means that only glucose was added.

Pellagra also results from a deficiency of nicotinamide, which is synthesized from tryptophan. Nicotinamide nucleotides are neither oxidized nor reduced during which step of cellular respiration? A.Glycolysis B.Chemiosmosis C.Citric acid cycle D.Electron transport chain

The answer is B because NAD+ is the oxidized form, and NADH is the reduced form of nicotinamide adenine dinucleotide. NAD+ is neither reduced to form NADH, nor is NADH oxidized to form NAD+ specifically during chemiosmosis.

Which mechanism best describes how P-gp facilitates drug resistance? A.P-gp binds to antitumor drugs in the presence of ATP and degrades the drugs. B.P-gp serves as a pump and uses active transport to move antitumor drugs outside the cell. C.P-gp prevents the entry of anti-tumor drugs into the cell. D.P-gp causes increased membrane permeability, which causes antitumor drugs to exit the cell.

The answer is B because P-gp is an ABC transporter protein, which uses ATP to actively transport antitumor drugs out of the cell.

The formation of which reaction products is increased in MBCD-treated MDR cells after exposure to 100 µM cholesterol? A.AMP + ADP B.ADP + Pi C.ATP + Pi D.ADP + ATP

The answer is B because at 100 µM cholesterol in the depleted MDR cells, ATPase activity, which is ATP hydrolysis activity, is completely restored. ATP hydrolysis produces ADP and inorganic phosphate.

The source of the phosphate groups that are added to rhodopsin is: A.arrestin. B.rhodopsin kinase. C.ATP. D.all-trans-retinol.

The answer is C because the requirement of ATP for kinase activity implies the phosphate groups come from ATP. All the other responses do not have phosphate to transfer.

In addition to glucose, what other monosaccharide is part of the UDP-glucose structure? A.Xylose B.Fructose C.Ribose D.Arabinose

The answer is C because UDP contains uridine, which is a nucleic acid used in RNA. The structure of RNA contains ribose.

Which reaction is catalyzed by LipA? A.ATP hydrolysis B.Peptide bond cleavage C.Hydrolysis of triacylglycerides D.Transfer of carboxyl groups

The answer is C because of the fact that LipA is a lipase, which means it must hydrolyze fatty acids.

When concentrated urine is being produced, in which of the following regions of the kidney will the glomerular filtrate reach its highest concentration? A.Proximal convoluted tubule B.Distal convoluted tubule C.Cortical portion of the collecting duct D.Medullary portion of the collecting duct

The answer is D because glomerular filtrate is most concentrated in the medullary portion of the collecting duct, compared to the other kidney structures listed.

What is the function of the Na+K+ ATPase during a neuronal action potential? A.Stimulation of the action potential B.Depolarization of the membrane C.Hyperpolarization of the membrane D.Restoration of the resting potential

The answer is D because the Na+K+ ATPase functions to restore the resting membrane potential by moving the ions against their concentration gradients.

Assuming no mutations to the signaling pathway described in the passage, what event directly activates CARD11? A.Proteolytic cleaveage B.GTP binding C.Membrane association D.Phosphorylation

The answer is D because the protein that activates CARD11 is PKC, which is a kinase. This implies that phosphorylation is the activating event.

An RNA molecule has 1500 bases. What is the maximum number of amino acids it can encode? A.500 B.1000 C.1500 D.4500

The answer to this question is A because each amino acid is encoded by three bases, so 1500/3 is 500, which is the maximum of amino acids the RNA molecule can encode.

Which statement provides the LEAST likely explanation for why the researchers dropped the highly anxious participants from their sample? A.Ethical committees do not allow research with participants who may have psychological disorders. B.If all the anxious participants get assigned to the stress condition, this may pose a confounding variable. C.Highly anxious participants are not representative of the population and may reduce the generalizibility of the findings. D.If all the anxious participants get assigned to the control condition, this may lead to a false rejection of the experimental hypothesis.

The answer to this question is A because it is ethically practicable to conduct research on people who score high on anxiety. However, having outliers accumulate in the experimental or control condition can lead to confounding or to errors in statistical inference. It may also limit the generalizability of the results.

The initial filtration step in the glomerulus of the mammalian kidney occurs primarily by: A.passive flow due to a pressure difference. B.passive flow resulting from a countercurrent exchange system. C.active transport of water, followed by movement of electrolytes along a resulting concentration gradient. D.active transport of electrolytes, followed by passive flow of water along the resulting osmolarity gradient.

The answer to this question is A because the initial filtration in the glomerulus occurs as blood pressure forces the fluid from the glomerulus into the lumen of Bowman's capsule.

Which measurement unit CANNOT be used to express power? A.kg•m2 •s2 B.J•s-1 C.ft•lb •s-1 D.W

The answer to this question is A because the measurement unit of power is watt, defined as J/s = ft•lb/s = kg•m2/s3.

Compared to the simultaneous condition, the serial condition of the experiment would be more likely to cause: A.a primacy effect. B.a state dependency effect. C.a misinformation effect. D.a dual-coding effect.

The answer to this question is A because, in the serial condition, the objects are presented to the participants one at a time, which might result in better memory for events at the beginning of the series (a primacy effect).

When a 0.1 M H2SO4 solution is added to pulverized blackboard chalk, the following reaction takes place. CaCO3(s) + H2SO4(aq) --> CaSO4(s) + CO2(g) + H2O(l) At 25°C, the reaction is spontaneous and has: A. negative ΔG° and positive ΔS°. B. negative ΔG° and negative ΔS°. C. positive ΔG° and negative ΔS°. D. positive ΔG° and positive ΔS°.

The answer to this question is A because spontaneous reactions must have a negative ΔG°, and reaction that create gaseous products from solids and liquids must have a positive ΔS°.

Gamma decay occurs when a nucleus emits: A.a photon. B.a proton. C.a neutron. D.an electron.

The answer to this question is A because, by definition, gamma decay means the emission of photons by the nucleus.

Ethanol may be metabolized to acetic acid, then condensed with a coenzyme to form acetyl coenzyme A. Acetyl coenzyme A may then participate in: A.the Krebs (citric acid) cycle. B.glycolysis. C.electron transport. D.oxidative phosphorylation.

The answer to this question is A, because acetyl coenzyme A is the main input of the citric acid cycle.

Which of the following statements gives the most fundamental reason why ornithine is unlikely to be found in proteins synthesized in vivo? A.There is no codon for it in the standard genetic code. B.It cannot form a peptide bond. C.It is not available in the diet. D.It has a net positive charge in aqueous solution.

The answer to this question is A, because without a corresponding codon in the genetic code, an amino acid cannot take part in the process of protein translation from an mRNA transcript.

Which effect is LEAST likely to occur with the process of gentrification? A.Development of affordable housing B.Increased neighborhood stratification C.Displacement of lower-income residents D.Expanded tax base for local government

The answer to this question is A, development of affordable housing. Gentrification is the reinvestment in lower income neighborhoods in urban areas, which results from the influx of more affluent groups. With the arrival of more affluent residents, housing demand increases and generally results in a decrease of affordable housing for lower income residents. In contrast, the other response options present more likely outcomes of gentrification.

Which statement does NOT identify an aspect of the concept of assimilation? A.Assimilation is the influence that cultural changes have on an individual's health. B.Assimilation is the process of cultural adaptation that results from geographic mobility. C.Assimilation occurs when individuals adopt the cultural norms of a dominant culture. D.Assimilation occurs when individuals relinquish the cultural norms of their childhood.

The answer to this question is A. Assimilation is related to the process of social integration and generally refers to when new members adopt the main elements of a culture. The other response options each provide an aspect of the definition of assimilation (cultural adaptation, adopting new norms, and relinquishing old norms). However, the answer does NOT describe assimilation. Instead, it speculates about a possible consequence of assimilation.

How will the rate of a catalyzed reaction be affected if the solid catalyst is finely ground before it is added to the reaction mixture? A.The rate will be faster because a greater mass of catalyst will be present. B.The rate will be faster because a greater surface area of catalyst will be exposed. C.The rate will be slower because the fine catalyst particles will interfere with product formation. D.The rate will remain the same because the mass of catalyst will be the same.

The answer to this question is B because grinding a heterogeneous catalyst increases the amount of catalyst available to the reaction and therefore increases its rate.

The production of a variety of opsins functions to: A.increase sensitivity to low light. B.enable the detection of different colors. C.ensure fast recovery of 11-cis-retinal after exposure. D.increase refractive index of the eye lens.

The answer to this question is B The wavelength of light absorbed by a molecule depends on its structure, and so the production of a variety of structurally related opsins functions to enable the detection of different colors.

A certain bacterium was cultured for several generations in medium containing 15N, transferred to medium containing 14N, and allowed to complete two rounds of cell division. Given that the bacterium's genome mass is 5.4 fg when grown in 14N media and 5.5 fg when grown in 15N medium, individual bacteria with which of the following genome masses would most likely be isolated from this culture? A.5.4 fg only B.5.4 fg and 5.45 fg C.5.4 fg and 5.5 fg D.5.45 fg only

The answer to this question is B because DNA replication is semi-conservative. Therefore, after the first round of cell division the genome mass in each bacterium will be 5.45 fg (one DNA strand will contain 15N and the other strand 14N). Following the second round of cell division, half of the bacteria will have a genome mass of 5.4 fg (14N exclusively) and the other half a mass genome of 5.45 fg (14N in one DNA strand and 15N in the other).

A researcher wants to gain insight into how a social movement organization presents its beliefs to the general public in an effort to attract new members. Which methodological approach would be most appropriate for this study? A.Observing meetings about the organization's marketing budget B.Conducting a content analysis of the organization's website C.Surveying organization members about their beliefs D.Randomly sampling the general population about social movement organizations

The answer to this question is B because conducting content analysis of online information would provide data on how the social movement organization frames issues to the broader public. Content analysis is a sociological method that is used to make inferences about communication. As applied to studying a website, content analysis could help determine which beliefs the organization publically emphasizes. In contrast, the methods in the other response options would not specifically generate data about the public presentation of beliefs.

The equilibrium BaCrO4(s) Ba2+(aq) + CrO42-(aq) exists in a saturated aqueous solution of BaCrO4. Dissolution of Na2CrO4 in a saturated aqueous BaCrO4 solution would: A.have no effect on the position of this equilibrium. B.shift this equilibrium left. C.shift this equilibrium right. D.shift this equilibrium first right and then left.

The answer to this question is B because dissolution of Na2CrO4 would introduce the common ion, CrO42-, which would reduce the solubility of BaCrO4 due to the common ion effect.

If adults performed the block tapping task as a measure of working memory capacity, they would most likely replicate sequences of what length correctly? A.9 plus or minus 1 B.7 plus or minus 2 C.10 plus or minus 1 D.16 plus or minus 2

The answer to this question is B because early memory research with adults led to the belief that the capacity of working memory was between 5 and 9 items. The so-called "magic number 7 ± 2" was the name George Miller (1956) gave to this estimate.

Which approach does NOT measure the activity of the Na+K+ ATPase? A.Measuring the rate of ATP hydrolysis B.Measuring the free energy of the ion transport C.Measuring the rate of ADP production D.Measuring the change in ion concentration within the liposome

The answer to this question is B because free energy is a thermodynamic quantity and is NOT a kinetic property. Activity is a kinetic property.

Despite the effects of diabetes, the brains of diabetic patients still receive adequate nourishment. This is most likely because the brain uses: A.less glucose than do other body tissues. B.insulin-independent transporters for the uptake of glucose. C.fatty acids for energy instead of glucose. D.insulin-dependent transporters for the uptake of glucose.

The answer to this question is B because glucose is the main fuel for brain cells, and since the brains of diabetic patients receive an adequate amount of glucose, the brain must use an insulin-independent mechanism for glucose uptake

Sound-induced vibrations depolarize hair cells of the cochlea by opening ion channels that are gated in what way? A.Chemically B.Mechanically C.Electrically D.Synaptically

The answer to this question is B because hair cells of the cochlea are specialized mechanoreceptors.

In general, telomeres are NOT important to bacterial cells because most bacterial chromosomes: A.do not replicate. B.are circular. C.replicate quickly and efficiently. D.are composed of single-stranded DNA.

The answer to this question is B because in most prokaryotes, DNA is circular and therefore does not have the repetitive, non-coding nucleotide sequences called telomeres that occur in eukaryotic DNA.

Interoceptive awareness involves sensitivity to increases in the activity of the: A.reticular activation system. B.autonomic nervous system. C.limbic system. D.somatic nervous system.

The answer to this question is B because interoceptive awareness is described as increased sensitivity to internal bodily sensations, such as heartbeat and blood pressure, which are regulated by the autonomic nervous system.

Which type of memory would most likely be activated first during the computer task, before working memory is activated? A.Implicit memory B.Sensory memory C.Short-term memory D.Episodic memory

The answer to this question is B because seeing squares light up on the computer screen would register with visual sensors, forming a sensory memory before working memory is activated.

Harlequin ichthyosis, a rare genetic disorder, causes the skin to become thick and scaly. Flaking skin behind the eyelids of individuals with this condition is most likely to damage which structure of the eye? A.Choroid B.Cornea C.Lens D.Retina

The answer to this question is B because the cornea is in direct contact with the eyelid and is thus most susceptible to damage from flaking skin.

Researchers hypothesize that experiencing a certain scent while viewing paintings of flowers will influence the viewers' ratings of attractiveness of the paintings. Researchers dispensed a floral scent in one room with paintings and no scent in another room with paintings. Which statement is accurate regarding the study? A.The study's design is an observational study since viewers were observing paintings and noting their ratings. B.The independent variable is the presence or absence of scent in the rooms. C.The dependent variable is the amount of time viewers spent observing paintings. D.Informing viewers ahead of time whether a scent was absent or present in the room is important for accurate ratings.

The answer to this question is B because the presence or absence of scent in the rooms is the manipulated variable.

What are the hybridization states of the carbon atoms involved in the conversion of trans to cis retinal? A.sp B.sp2 C.sp3 D.sp3d

The answer to this question is B because the reacting carbon atoms are both central to AX3 systems with 3 bonded atoms and no lone pairs. The preferred geometry for such a system is trigonal planar and the hybridization scheme that facilitates this geometry is sp2.

Glucose transporter proteins in the liver do not require the presence of insulin to facilitate the uptake of glucose. However, insulin does stimulate the first step in the glycolytic pathway within the liver. Therefore, in liver cells, insulin most likely: A.hinders glucose uptake by increasing the cellular concentration of glucose. B.aids glucose uptake by decreasing the cellular concentration of glucose. C.hinders glucose uptake by using the ATP needed by the glucose transporter proteins. D.aids glucose uptake by providing the ATP needed by the glucose transporter proteins.

The answer to this question is B because the stem states that insulin stimulates the first step in the glycolytic pathway in the liver, thus decreasing the cellular concentration of glucose. This results in increased uptake of glucose to maintain the cellular concentration of glucose.

In a study, each trial involves administering a drop of lemon juice to the participant's tongue and measuring the participant's level of salivation. As more trials are conducted, the researcher finds that the magnitude of salivation declines. After a certain point, the researcher switches to administering lime juice. This researcher is most likely studying which process? A.Sensory perception B.Habituation and dishabituation C.Stimulus generalization in classical conditioning D.Conditioned responses in classical conditioning

The answer to this question is B because the study involves reduced responding to a repeating stimulus, which is best described as habituation. The researcher then changes the stimulus, which will likely lead to dishabituation.

According to Erikson's theory of psychosocial development, which group of participants are most likely dealing with the challenges of generativity versus stagnation? Adolescents Young adults Adults A.I only B.III only C.I and II only D.II and III only

The answer to this question is B because, in Erikson's theory, the generativity versus self-absorption crisis occurs in mid-adulthood. Thus, the adults compose the only age group that is likely to experience this stage.

The ornithine decarboxylase reaction has been studied extensively by biomedical researchers. The most likely reason for the interest of these researchers is that the reaction is: A.a good source of ornithine. B.an early event in cell division. C.unique to mammals. D.one in which a chiral reactant is converted to an achiral product.

The answer to this question is B, because the passage states that in mammals and other organisms ornithine decarboxylase participates in early events in cell division.

Which type of design is LEAST appropriate for research on residential segregation? A.Ethnographic methods B.Experimental methods C.Quantitative methods D.Survey methods

The answer to this question is B, experimental methods. Due to ethical concerns, as well as practical considerations, experimental research is generally not available for studying residential segregation. Except under special circumstances, it would be unethical and impractical to randomly assign participants to groups in which segregation is manipulated as an independent variable. In contrast, the other response options present more acceptable methods.

An immigrant teen starts to reject the ethnic customs of his family and instead identifies himself as an American by dressing, speaking, and acting in ways that are associated with American culture. In this scenario, Americans become which type of group for the teenager? A.Assimilated group B.Reference group C.Majority group D.Peer group

The answer to this question is B, reference group. Individuals often emulate the attitudes and behaviors of groups that they admire and would like to join. Sociologists refer to these as reference groups, which are important for self-evaluation and identity formation. The scenario in the question suggests that the immigrant teen constructs an identity, through modifying his behaviors (dressing, speaking, and acting), to match that of the broad cultural group of "American."

Often utilized when studying communicable diseases, which type of analysis maps the series of relationships among a set of individuals? A.Social support analysis B.Social network analysis C.Social stratification analysis D.Social reproduction analysis

The answer to this question is B, social network analysis. Used in some epidemiological studies, social network analysis is the method described in the question. It involves the mapping of social relationships among individuals.

Which research methodology involves the extended, systematic observation of a complete social environment? A.Comparative methods B.Ethnographic methods C.Experimental methods D.Survey methods

The answer to this question is B. Ethnographic methods are accurately described in the question as involving systematic observation of a complete social environment.

Each of the following equations shows the dissociation of an acid in water. Which of the reactions occurs to the LEAST extent? A. HCl + H2O → H3O+ + Cl− B. HPO42− + H2O → H3O+ + PO43− C. H2SO4 + H2O → H3O+ + HSO4− D. H3PO4 + H2O → H3O+ + H2PO4−

The answer to this question is B. HPO42− has a high negative charge and so dissociation of it will occur to the least extent.

Which of the following most likely will occur if a homogeneous catalyst CANNOT be separated from the products at the end of a reaction? A.The catalyst will become heterogeneous. B.The products will be contaminated. C.The reaction will not occur. D.The reaction rate will speed up.

The answer to this question is B. If a homogeneous catalyst cannot be separated from the products at the end of a reaction then the products will be contaminated with the catalyst.

When a strip of Zn is placed in a beaker containing 0.1 M HCl, H2(g) evolves. If a strip of Al is placed in a beaker containing 0.1 M HCl, does H2(g) evolve? A.Yes; Al is reduced and H+(aq) is oxidized. B.Yes; Al is oxidized and H+(aq) is reduced. C.No; Al is reduced and Cl-(aq) is oxidized D.No; Al is oxidized and H2O(l) is produced.

The answer to this question is B. Since a new solid forms when Al(s) is mixed with Zn2+(aq), it is reasonable to assume that Al(s) is more susceptible to oxidation than Zn(s).

What is the total number of fused rings present in a steroid? A.1 B.2 C.4 D.6

The answer to this question is C because a steroid is defined by its fused 4-ring structure.

A researcher replicates the experiment with the addition of a physical stressor to the first phase of the experiment. According to Selye's general adaptation syndrome, this change is: A.necessary, because humans respond differently to different types of stressors. B.necessary, because physical stressors cause avoidance-avoidance conflict, whereas social stressors cause approach-approach conflict. C.unnecessary, because the human stress response is not specific to the type of stressor. D.unnecessary, because both physical stressors and social stressors cause avoidance-avoidance conflict.

The answer to this question is C because, according to Selye's general adaptation syndrome, people's response to various stressors is similar. Selye's theory does not make claims about avoidance-avoidance or approach-approach conflicts.

The statement that the ornithine decarboxylase assay is highly specific means that it: A.requires radioactive ornithine of high specific activity. B.generates diaminobutane of high specific activity. C.an distinguish ornithine decarboxylase activity from the many other enzymatic reactions in a cell. D.can measure the small amount of ornithine present in a cell.

The answer to this question is C because enzymes such as ornithine decarboxylase are highly specific both in the reactions that they catalyze and in their choice of substrates and therefore catalyze only a single chemical reaction or occasionally a set of closely related reactions.

During the production of insulin, the translated polypeptide is cleaved into the mature form and secreted from the cell. The cleavage most likely takes place in which of the following locations? A.Nucleus B.Ribosomes C.Endomembrane system D.Cytoplasm

The answer to this question is C because secreted proteins such as insulin are cleaved into mature form within endomembrane system.

A student is investigating the bystander effect and receives permission to conduct the study in the school cafeteria. What is a potential dependent variable that can be measured by the student? A.The time of day that the experiment is carried out B.The number of people present in the cafeteria C.The amount of time a student takes to assist another student in distress D.The temperature in the cafeteria

The answer to this question is C because the amount of time a student takes to help another student in the cafeteria is the only option that represents a measurable indication of the helping response, which is the dependent variable in studies of the bystander effect.

The researchers collected data on participants' heart rates and electrical skin conductance at different stages of the experiment. Which pattern would verify that the independent variable had the intended effect? Heart rate and skin conductance are higher: A.for all the participants before they participate in the first phase of the experiment compared to after. B.for the participants who chose to share $120 compared to those who chose to walk away with $30. C.for the participants in the stress group than for the participants in the control group. D.for all the participants before they participate in the second phase of the experiment compared to after.

The answer to this question is C because the independent variable in the study was exposure to a stressor. The stress group (who expected to give a presentation) should differ from the control group (who did not expect to give a presentation). The stress response is associated with elevated heart rate and skin conductance. Therefore, the stress condition should lead to higher scores on these variables.

Exercise promotes the insulin-independent uptake of glucose in working skeletal muscles. Given this, regular exercise would most likely reduce blood glucose levels in patients with which type(s) of diabetes? A.Type 1 only B.Type 2 only C.Both Type 1 and Type 2 D.Neither Type 1 nor Type 2

The answer to this question is C because the stem states that exercise promotes the insulin-independent uptake of glucose and thus it will reduce blood glucose levels both in patients with impaired production of insulin (type 1 diabetes), and in patients who do not respond to insulin (type 2 diabetes).

Why is the velocity of blood flow slower in capillaries than in arteries? A.Capillary walls are more elastic than arterial walls. B.Capillaries have less resistance to blood flow than arteries. C.The total cross-sectional area of capillaries exceeds that of arteries. D.Blood pressure is higher in the capillaries than in arteries.

The answer to this question is C because the high number of capillaries in the body means that the total cross-sectional area of these vessels is larger than any other vessel type in the circulatory system. This causes the velocity of the blood to decrease.

Which of the following diagrams does NOT illustrate a function of an automobile catalytic converter?

The answer to this question is C because this response depicts a chemical transformation that is NOT catalyzed by an automobile's catalytic converter.

The term "ideal gas" refers to a gas for which certain assumptions have been made. Which of the following is such an assumption? A.The law PV = nRT2 is strictly obeyed. B.Intermolecular molecular forces are infinitely large. C.Individual molecular volume and intermolecular forces are negligible. D.One gram-mole occupies a volume of 22.4 L at 25°C and one atmosphere pressure.

The answer to this question is C since a property of an "ideal" gas is that it is composed of particles that have negligible volume and do not exert intermolecular forces.

The most rapid rate of gluconeogenesis will most likely occur in the body when: A.blood glucose levels are high. B.cortisol release is inhibited. C.the body's stores of carbohydrates are low. D.the body's stores of proteins are low.

The answer to this question is C, because gluconeogenesis is the pathway for the synthesis of glucose from other metabolic compounds and thus it is activated when the body's stores of carbohydrates are low.

During the initiation of muscle contraction, mysosin binds actin after troponin binds to which ion? A.H+ B.K+ C.Na+ D.Ca2+

The answer to this question is D because during the initiation of muscle contraction, free Ca2+ in the cytosol binds to troponin, which pulls tropomyosin away from actin's myosin-binding site. This allows myosin to bind to actin. The other ions are not involved in actin-myosin binding.

Colchicine is a drug that prevents the formation of microtubules. Which of the following mitotic processes would NOT occur after exposure to this drug? A.Formation of new cell plate B.Replication of DNA during interphase C.Breakdown of the nucleolus during prophase D.Movement of the chromosomes toward opposite poles of the cell during anaphase

The answer to this question is D because microtubules function in chromosomal movements in cell division.

After initially learning to ride a bike, riding a bike becomes easy for an individual. Each time the individual rides a bike thereafter, what type of memory is being used? A.Semantic B.Episodic C.Explicit D.Procedural

The answer to this question is D because procedural memory refers to memory for the performance of particular types of action.

During an action potential, the movement of sodium ions into a neuron causes the neuronal membrane to do which of the following? A.Inactivate B.Hyperpolarize C.Repolarize D.Depolarize

The answer to this question is D because the movement of sodium ions into a neuron during an action potential results in opening of more voltage-gated sodium channels, causing further depolarization.

Suppose squares 1, 2, 3, and 4, moving from left to right at the top of the grid light up and some participants think "top row lights up first," to remember the sequence. Which process is most likely being used to enhance working memory capacity? A.Divided attention B.Priming C.Automatic processing D.Chunking

The answer to this question is D because the participants are organizing single unit squares into larger rows to help them hold more information in working memory. This encoding process, which is referred to as "chunking," requires effortful processing and therefore involves selective attention and conscious effort.

Which of the following types of orbitals of the central atom are involved in bonding in octahedral compounds? A.sp B.sp3 C.p D.d2 sp3

The answer to this question is D because octahedral compounds have six σ bonds and no lone pairs. According to valence bond theory, the central atom requires the hybridization of six atomic orbitals, d2sp3.

The environment of the retinal binding site is most likely: A.hydrophilic. B.positively charged. C.negatively charged. D.hydrophobic.

The answer to this question is D because retinal is composed of mainly carbon and hydrogen, making it largely hydrophobic.

Assume that K and M are two unlinked genes that affect hearing. The dominant K allele is necessary for hearing, and the dominant M allele causes deafness regardless of the other genes present. Given this, what fraction of the offspring of parents with the genotypes KkMm and Kkmm will most likely be deaf? A.1/4 B.3/8 C.1/2 D.5/8

The answer to this question is D, because among the offspring of KkMm and Kkmm parents, the ones who lack a dominant K allele (necessary for hearing), or carry a dominant M allele (causes deafness) are deaf. Based on the Punnett square analysis, 10 out of 16 or 5/8 of all offspring are likely to be deaf.

What volume of a 0.120 M CaI2 solution would contain 0.078 mol of the solute? A.35.0 mL B.65.0 mL C.350 mL D.650 mL

The answer to this question is D. In order to obtain the volume of solution necessary to provide a given amount of solute in moles, one needs to take the number of moles and divide by the solution concentration in molarity.

Sociologists describe the health benefits of social support as resulting from all of the following EXCEPT: A.friends and family members that help patients adhere to medical treatment. B.friends and family members that help patients to reduce harmful behaviors. C.social relationships that help individuals to cope with stressful events. D.social activities that help individuals to avoid loneliness and boredom.

The answer to this question is D. Social support refers to social network ties (friends, family, and other relationships) that provide an individual with various types of assistance, which are associated with improving health or reducing harm. All of the response options except for D address the health benefits of networks. The correct answer identifies activities rather than relationships.

Enzymes alter the rate of chemical reactions by all of the following methods EXCEPT: A.co-localizing substrates. B.altering local pH. C.altering substrate shape. D.altering substrate primary structure.

The answer to this question is D. The primary structure of a protein substrate is the amino acid sequence of the protein. Enzymes cannot alter primary structures of protein, but can colocalize substrates, alter local pH, and alter substrate shape.

Participants in a weight-loss program agree to have their body mass index (BMI) measured to track their progress in the program. Among a sample of 72 participants, the mean BMI is 30 and the median BMI is 25. Which statement provides an accurate description of the sample? A.More participants had a BMI over 25 than a BMI under 25. B.The majority of participants had a BMI between 25 and 30. C.Half of the participants had a BMI over 25 and half had a BMI under 25. D.More outliers among the participants had a BMI under 25 than a BMI over 30.

The answer to this question is option C because the median splits the sample distribution of values in half. Thus, a median of 25 can be described as half the sample having a BMI over 25 and half having a BMI under 25. The statements in the other options do not correctly describe the sample data in the question

Which of the following quantum number sets is possible? A. n=2; l=2; m1=1; ms= +1/2 B. n=2; l=1; m1=-1; ms= +1/2 C. n=2; l=0; m1=-1; ms= -1/2 D. n=2; l=0; m1=1; ms= -1/2

The azimuthal quantum number l cannot be higher than n-1, ruling out choice (A). The ml number, which describes the chemical's magnetic properties, can onlt an an integer calue between -l and l. It cannot be equal to +/-1 if l=0; this would imply that an s orbital has three subshells (-1, 0, and 1) when we know it can only have one. This rules out choices (C) and (D).

Research findings indicate higher levels of patient trust and physician attention in race-concordant physician-patient relationships because: A.both the physician and patient belong to the same in-group. B.the physician and patient share similar levels of status. C.of racial hostility present in race-discordant relationships. D.of the lack of role strain experienced by the physician.

The correct answer is A. Members of in-groups share common backgrounds and similar identities, which generally result in high levels of trust among in-group members. Racial/ethnic identity can be an important characteristic of an in-group, resting in a shared culture, language, or community.

Which term refers to closed status positions that hinder social mobility? A.Class systems B.Caste systems C.Economic systems D.Financial systems

The correct answer is B. Caste systems describe closed stratification systems that do not allow for social mobility.

Neighborhood-level socioeconomic inequalities are most likely to affect physician-patient interactions through which phenomenon? A.Status hierarchies rely on socioeconomic differences above other master status characteristics. B.Physical boundaries create social boundaries and closed networks which develop their own cultures. C.Socioeconomic inequalities create role conflict in physicians who are trained to treat all patients equally. D.The physician-patient relationship is not subject to the same external forces that govern other relationships.

The correct answer is B. Neighborhoods that are segregated by socioeconomic status create increased social distance to the extent that people from different backgrounds have trouble relating to each other's experiences. The correct answer succinctly explains this by referring to closed networks and social boundaries that exist across neighborhoods.

What is the advantage of including a specially trained cultural liaison as opposed to training physicians in cultural competency? A.Communicating across racial and cultural lines is difficult and requires a trained specialist to do it effectively. B.Physicians are inherently resistant to cultural training because of their medical biases. C.Adding a cultural liaison creates a triadic group, a more stable arrangement than a dyadic group. D.Larger groups facilitate the transfer of information more effectively than smaller groups.

The correct answer is C. From the perspective of basic group dynamics in sociology, larger groups are generally considered more stable but less intimate, whereas smaller groups are usually considered less stable but more intimate. Dyads, two-person groups such as the physician-patient group, are unstable because either party can break the single social tie. The triad, three person groups such as the physician-patient-cultural liaison group, is considered relatively more stable because of the additional social ties.

Because cultural liaisons work closely with physicians and are advocates for patients, which phenomenon are cultural liaisons most likely to experience? A.Network redundancy because of the amount of people with whom they come into contact B.Homogeneity because they will see the same patients over and over C.Role strain because of their collegiality with physicians and their responsibility to patients D.Role conflict because of their status as a non-medical professional in a clinic setting

The correct answer is C. Role strain is the tension that results from competing demands within the context of a single role. As applied to the question, the tension that the cultural liaison experiences is between different responsibilities within the same occupational role. If the tension existed between different roles that a single individual held, then it would be considered role conflict.

Which of the following organelles is surrounded by a single membrane? A. Lysosomes B. Mitochondria C. Nuclei D. Ribsomoes

A Lysomes are vesicular organelles that digest material using hydrolytic enzymes. They are surrounded by a single membrane. Both mitochondira and nuclei are surrounded by double membranes, eliminating choices (B) and (C). Ribosomes must not be surrounded by membranes because they are found not only in eukaryotes, but also in prokaryotes, which lack any membrane bound oragnelles, eliminating choice (D).

Which is true of teaching an animal a complicated, multistage behavior? I. The individual parts of the behavior should not run counter to the animals natural instincts II. The behaviors must be tied to a food reward of some kind. III. Rewarding individual parts of the behavior on their own interferes with reinforcement of the entire behavior. A. I only B. I and III only C. II and III only D. I, II, and III

A. Complicated, multistage behaviors are typically taught through shaping, so statement III must not be part of the correct answer. Reinforcers do not necessarily need to be food-based, and instinctive drift can interfere with learning of complicated behaviors; therefore, only statement I is correct.

EEG waveforms during REM sleep most resemble which of the following states of consciousness? A. Alertness B. Slow-wave sleep C. Stage 1 sleep D. Meditation

A. EEG during REM is composed mainly of beta weaves, which are present during alertness. SWS, choice (B), consists mainly of delta waves, which are not typically present during REM sleep. Stage 1 sleep, choice (C), consists mainly of theta waves. Meditation, choice (D), is quieting of the mind, and consists mainly of slow alpha waves and theta waves.

A bacterial cell is noted to be resistant to penicillin. The bacterium is transferred to a colony that lacks the fertility factor, and the rest of the colony does not become resistant to penicillin. However, the penicillin-resistant cell has also started to exhibit other phenotypic characteristics, including secretion of a novel protein. Which of the following methods of bacterial recombination is NOT likely to account for this change? A. Conjugation B. Transformation C. Transduction D. Infection with a bacteriophage

A. A bacterial cell that does not rapidly cause a phenotypic change in the rest of the colony is likely no F+, meaning that this cell is not able to form a sex pilus for conjugation. The expression of new phenotypic characteristics indicate that this bacterium may have acquired some quantity of genetic information, which can be acquired by either transformation, choice (B), or transduction (which occurs via bacteriophage infection), choices (C) and (D).

After infection of a cell, a viral particle must transport itself to the nucleus in order to produce viral proteins. What is the likely genomic content of the virus. A. Double-stranded DNA B. Double-stranded RNA C. Positive-sense RNA D. Negative-sense RNA

A. A virus that requires transport to the nucleus in order to produce viral proteins requires use of nuclear RNA polymerase in order to create mRNA that can be translated to protein. Only DNA viruses require this transport before any protein can be synthesized.

Researchers repeatedly startle a participant with a loud buzzer. After some time, the participant stops being startled by the buzzer. If the researchers interrupt the study with the sound of pans banging together, which of the following would likely be observed? A. Increased startle response to the buzzer B. Decreased startle response to the buzzer C. No change in res ponce to the buzzer D. Generalization to previously nonaversive stimuli

A. After a while, the participant became habituated to the sound of the buzzer. Introducing a new stimulus, such as the banging of pans, should dishabituate (resensitize) the original stimulus, causing a temporary increase in response to the sound of the buzzer.

A district attorney with an internal locus of control wins an important court trial. Which of the following best represents the lawyers attribution of the events? A. "I won because I made great arguments and had more experience than the defense." B. "I won because the jury was on my side from the beginning and believed in my arguments." C. "I won because the defense did not adequately present their side of the case." D. "I shouldn't have won because I don't deserve to be successful."

A. Because we know the lawyer has an internal locus of control, we expect her to believe that she is in control the the events that happen in her life. Both choices (B) and (C) attribute success to outside factors, representing an external locus of control. While choice (D) perhaps represents an attribution that could correlate to low self-esteem, it is not indicative of locus of control.

A stroke patient comprehends speech but cannot properly move her mouth to forms words. Which of the following brain areas is likely affected? A. Broca's Area B. Wernicke's area C. Arcuate fasiculus D. Superior temporal gyrus

A. Broca's area governs the motor function of language. A stroke that affects Broca's area will leave receptive language intact, but word formation will be affected. A stroke affecting Wernicke's area, choice (B), will make it so the individual is unable to comprehend speech. A stroke affecting the arcuate fasciculus, choice (C), will result in an inability to repeat words heard but spontaneous language production is intact. The superior temporal gyrus, choice (D), is where wernicke's area is located.

Seeking homeostasis to reduce an uncomfortable internal state is associated with which motivational theory? A. Drive reduction theory B. Instinct theory C. Arousal theory D. Incentive theory

A. Drive reduction theory is the theory that one will act to eliminate uncomfortable internal states known as drives. The body will push toward equilibrium, or homeostasis.

A college student strives for excellent grades and hopes to graduate with a better GPA than his brother. This type of motivation is considered: A. extrinsic motivation B. intrinsic motivation C. a primary drive D. a secondary drive

A. Due to the competitive nature of the motivation, this is considered extrinsic motivation. Extrinsic motivation is based on external conditions, including perceived reward or fear of punishment. In this case, the reward is beating his brother. There is no suggestion of an uncomfortable internal state or tension, which is an aspect of drives, eliminating choices (C) and (D).

Which of the following types of intelligence is NOT described by Gardner's theory of multiple intelligences? A. Fluid intelligence B. Body-kinesthetic intelligence C. Visual-spatial intelligence D. Linguistic intelligence

A. Fluid intelligence consists of problem-solving skills and is not on of Gardner's seven multiple intellengences. Gardner's theory list linguistic, logical-mathematical, musical, visual-spatial, bodily-kinesthetic, interpersonal and intraperaonl intelligences.

If the amount of acetylcholinesterase, an enzyme that breaks down acetylcholine, is increased, which of the following would likely be the result? A. Weakness of muscle movements B. Excessive pain or discomfort C. mood swings and mood instability D. auditory and visual hallucinations

A. If there were increased amounts of acetylcholinesterase, more acetylcholine would be degraded, lowering acetylcholine levels in the body. Low levels of acetylcholine would result in weakness or paralysis of muscles. Pain, choice (B), would result if one was injured and endorphines were found in low levels. Mood swings, choice (C), would be a result of varying levels of serotonin. Hallucinations, choice (D), have been seen to result from high levels of dopamine.

Matt and Cati discuss the reasons why they avoid driving above the speed limit. Matt says that he wants to avoid a traffic fine, while Cati says that speeding is dangerous and, if everyone did it, there would be more accidents and people would get hurt. According to Kohlberg, which of the following describes the phases of moral reasoning demonstrated by Matt and Cati, respectively? A. Preconventional; conventional B. Preconventional; postconventional C. Conventional; preconventional D. Postconventional; conventional

A. Matt's reasoning reflects a desire to avoid punishment, which reflects stage one in Kohlberg's preconventional phase (obedience). Cati's reasoning takes into account social order, reflecting stage four in the conventional phase (law and order).

Mitochondrial DNA is: I. Circular II. Self-replicating III. Single-stranded A. I only B. II only C. I and II only D. I, II and II

A. Mitochondria are thought to have evolved from an anaerobic prokaryote engulfing an aerobic prokaryote and establishing a symbiotic relationship; therefore, mitochondrial DNA, or mDNA, is likely to be similar to bacterial DNA. Both mDNA and bacterial DNA are organized into a single circular chromosomes of double-stranded DNA that can replicate during binary fission. Therefore, statements I and II are correct, while statement III is incorrect.

A researcher uses a partial-report procedure after presenting participants with an array of nine numbers for a fraction of a second. Which of the following is the most likely result of this procedure? A. The participant will be able to recall any of the rows or columns in great detail but only immediately after presentation. B. The participant will only be able to recall the first few numbers in the array due to the serial position effect. C. The participant will be able to recall approximately seven of the numbers for a few seconds following presentation of the stimulus. D. The participant will not be able to recall any of the numbers verbally, but will be able to draw the full array under hypnosis.

A. Partial-report procedures, in which the individual is asked to recall a specific portion of the stimulus, are incredibly accurate, but only for a very brief time. This is a method of studying sensory (specifically, iconic) memory. Both the serial position effect, choice (B), and the 7+/- rule, choice (C), are characteristics of short-term memory.

Chemicals that compel behavior after binding with chemoreceptors are known as: A. pheromones B. olfactory receptors C. somatostimuli D. papillae

A. Pheromones are the volatile chemicals given off by organisms that bind with olfactory chemoreceptors and influence behavior. It is debatable if pheromons serve a role in humans, but are known to affect foraging and sexual behavior in some animals.

Which of the following methods of encoding is most conductive to later recall? A. Semantic B. Visual C. Iconic D. Acoustic

A. Semantic encoding, or encoding based on the meaning of the information, is the strongest of the methods of encoding. Visual encoding, choice (B), is the weakest, and acoustic encoding, choice (D), is the intermediate between the two. Iconic memory, choice (C), is a type of sensory memory.

The ability to sense stimuli against one's own skin is known as: A. somatosensation B. kinesthetic sense C. vestibular sense D. chemoreception

A. Somatosensation refers to the various modalities of touch: vibration, temperature, and pain. Kinesthetic sense, choice (B), refers to the ability to tell where one's body is in space. Vestibular sense, choice (C), refers to the detection of linear and rotational acceleration in the middle ear. Finally, chemoreception, choice (D), refers to sensing chemicals in the environment.

The adrenal glands do all of the following EXCEPT: A. promote the flight-or-fight response via estrogen B. Produce stress responses via cortisol C. Produce both hormones and neurotranmitters D. Release estrogen in males and testosterone in females

A. The adrenal glands do promote the flight-or-fight response, but through epinphrine and norepinephrine, not estrogen. The adrenal cortex produces both estrogen and testosterone in both sexes, as mentioned in choice (D), thus serving as a source of estrogen in males and testosterone in females.

Which part of the eye is responsible for gathering and focusing light? A. Cornea B. Pupil C. Iris D. Retina

A. The cornea is responsible for gathering and focusing light. The iris and pupil, choices (B) and (C), are both involved in regulating the amount of light coming into the eye but not in focusing it. The retina, choice (D), transduces the light into electrical signals that are sent to the brain. The lens serves a similar function to the cornea and would be a valid answer ton this question.

While cleaning your house, you notice a large spider on the wall by your head and feel your heart rate jump up and your skin temperature grow warm. Which stage of stress response are you experiencing? A. Alarm B. Resistance C. Exhaustion D. Homeostasis

A. The initial reaction to stress, which is activation of the sympathetic nervous system, is the alarm stage of stress response.

What is the main function of the nucleolus? A. Ribosomal RNA synthesis B. DNA replication C. Cell division D. Chromosome Assembly

A. The nucleolus (not to be confused with the nucleus in general) is a dense structure within the nucleus where ribosomal RNA (rRNA) is synthesized.

Which of the following activities would most likely be completed by the right hemisphere of a left handed person? A. Finding a car in a parking lot B. Learning a new language C. Reading a book for pleasure D. Jumping rope with friends

A. The right hemisphere is usually the non-dominant hemisphere, even in left-handed individuals. Sense of direction is an ability of the non-dominant hemisphere. The other answer choices are all abilities attributed to the dominant hemisphere.

People from cultures around the world can identify which of the following emotions? A. Happiness, sadness, and surprise B. Happiness, anger, and apathy C. Sadness, anticipation, and happiness D. Excitement, anger, and disgust

A. The seven universal emotions are happiness, sadness, contempt, surprise, fear, disgust, and anger.

A woman is at a restaurant and orders a spicy entreée. After the first bite, she experiences burning in her mouth and becomes concerned that her food is too hot for her. The next few bites are similarly uncomfortable, but after a while the spiciness seems to subside somewhat, and by the end of the meal, she doesn't notice the spice level. The end of the meal experience is best described as: a. adaptation. b. signal detection. c. a difference threshold. d. pain perception.

A. The spicy food can be considered an extreme stimulus because it eclipses what the woman believes she can handle in terms of heat. However, after experiencing the stimulus over and over, the experience of spicy drops to barely perceptible. This is sensory adaption: a reduction in response to a stimulus over time.

Which of the following does not describe connective tissue cells? A. They account for most cells in the muscles, bones, and tendons B. They secrete substances to form the extracellular matrix C. In organs, they tend to form the stroma D. In organs, they provide support for epithelial cells.

A. While bones and tendons are composed predominantly of connective tissue cells, muscle tissue is considered a different tissue type. Other examples of connective tissue include cartilage, ligaments, adipose tissue, and blood. Connective tissue often secrets substances to form the extracellular matrix, such as collagen and elastin, eliminating choice (B). Choices (C) and (D) are essentially identical and can be be eliminated: in organs, connective tissue often forms the support structure for epithelial cells, called the stroma.

Which of the following activities occurs in the Golgi Apparatus? A. Synthesis of proteins B. Modification and distribution of proteins C. Breakdown of lipid and carbohydrates D. Production of ATP

B. The Golgi apparatus consists of a stack of membrane enclosed sacs. It receives vesicles and their contents from the endoplasmic reticulum, modifies them )through glycosylation, phosphorylation, and other mechanisms), repackaging them into vesicles, and distributes them to appropriate locations in the cell.

Which kind of conflict is associated with the LEAST amount of stress? A. approach-approach conflict B. Avoidance-avoidance conflict C. approach-avoidance conflict D. avoidance-escape conflict

Approach-approach conflict is one in which both results are good outcomes. While one must be chosen, neither choice results in a negative outcome: for example, choosing between two deserts. Avoidance-escape conflict, choice (D), is not a recognized for of conflict; these two terms are related to types of negative reinforces.

A high school student struggles consistently with math and feels that no matter how hard he studies, he "just doesn't get it." Which of the following is the most likely short-term result with respect to his ability to do math? A. Low self-esteem B. Low self-efficacy C. Learned helplessness D. An external locus of control

B. Because there is nothing in the question stem to suggest that this situation will fundamentally change this student's attitudes in the short term, choices (A) and (D) can be eliminated. Choice (C) is unlikely in the short term, as learned helplessness requires a repeated inability to have any effect on a situation over a long period of time and is much more severe, usually manifesting as depression. It is far more likely that the student will simply fell ineffective when it comes to math, which is low self-efficacy.

Which of the following would elderly individuals be most likely to have trouble recalling? A. The circumstances of meeting his or her significant other in college B. A doctor's appointment scheduled for 1:00 p.m. C. The names of the characters in his or her favorite television show D. That a library book needs to be returned when passing by the library on a morning walk

B. Elderly individuals have the most trouble with time-based prospective memory, which is remembering to do an activity at a particular time. Other forms of memory are generally preserved, or may decline slightly but less significantly than time-based perspective memory.

Each of the following is considered a part of a persons self-concept EXCEPT: A. the past self B. the ought self C. the future self D. self-schemata

B. Self-concept is defined as the sum of all the ways in which we see ourselves, including who we are, as in choice (D), who we were in the past, choice (A), and who we may become in the future, choice (C). The ought self, whole closely related to self-esteem, is our appraisal of how others see us, and is not a part of our self-concept.

Which of the following indicates the pattern of sleep stages during a complete sleep cycle early in the night? A. 1-2-3-4-1-2-REM B. 1-2-3-4-3-2-REM C. 4-3-2-1-2-3-REM D. 4-3-2-4-3-1-REM

B. Early in the evening, sleep cycles include deepening of sleep (stages 1-2-3-4), followed by lightening of sleep )Stages 4-3-2) and then REM, or just directly moving from Stage 4 into REM. Later in the evening, the cycle may be shortened as slow-wave sleep becomes less common.

A person suffers from food poisoning after eating a spoiled orange, and later finds that the smell of lemon and other citrus fruits make her nauseated. This is an example of: A. Aquisition B. Generalization C. discrimination D. negative reinforcement

B. Generalization is the process by which similar stimuli can produce the same conditional response. Here, the response to the taste and smell of oranges has generalized to that of all citrus.

Prolonged vitamin B12 deficiency can be associated with subacute combined degeneration of the spinal cord. Patients with this disease have difficulty walking because they lose the ability to feel where their feet are in space. This represents a loss of: A. Vestibular Sense B. Kinesthetic sense C. Parallel processing D. feature detection

B. Kinesthetic sense, or proprioception, refers to the ability to tell where body parts are in three-demensional space. The sensors for proprioception are found predominantly in the muscles and joints. Loss of vestibular sense, choice (A), would also cause difficulty walking, but this would be due to a sense of dizziness or vertigo, not an inability to feel one's feet.

Which language theory states that language development occurs due to preferential reinforcement of certain phonemes by parents and caregivers? A. Nativist theory B. Learning theory C. Social interactionist theory D. Neurocognitive theory

B. Learning theory, largely based on the work of B. F. Skinner, states that parents reinforce phonemes that sound most like their language, resulting in preferential preservation of these phonemes. Nativist theory, choice (A), posits a critical period during which language acquisition occurs. Social interactionist theory, choice (C), indicates that language develops via interaction with parents and caregivers as a desire of the child to communicate. Neurocognitive theory, choice (D), is concerned with the subjective experiences of dreaming and the physiology of dreaming

Which of the following fine motor tasks would one expect to see first in an infant? A. Grasping for objects with two fingers B. Following objects with the eyes C. Scribbling with a crayon D. Moving a toy from one hand to another

B. Motor skills tend to develop from the core towards the periphery. Following objects with the eyes occurs around four weeks of age. The other actions all require movements of the hand, which do not occur in an organized fashion until later.

Which of the following is/are true with regard to neurulation? I. The neural tube differentiates from endoderm II. The neural tube becomes the peripheral nervous system III. Neural crest cells migrate form their original site A. I only B. III only C. II and III only D. I, II, and III

B. Neurulation occurs when the notochord causes differentiation of overlaying ectoderm into the neural tube and neural crest cells. The neural tube ultimately becomes the central nervous system (brain and spinal cord), and the neural crest cells migrate to other sites in the body to differentiate into a number of different tissues. Thus, only statement III is true.

Which sensory receptors send signals in response to tissue damage? A. Chenoreceptors B. Nociceptors C. Osmoreceptors D. Photoreceptors

B. Nociceptors are important for pain sensation, which would be expected during tissue damage. Chemoreceptors, choice (A), responds to chemicals, whether volatile or aerosolized (olfaction) or dissolved (taste). Osmoreceptors, choice (C), respond to change in blood osmolarity, and photoreceptors, choice (D), responds to light.

Which of the following is an example of semantic memory? A. Having the ability to drive a car B. Knowing the parts of a car engine C. Remembering the experience of learning to drive D. Associating a car with other vehicles in a semantic network

B. Semantic memory is the category of long-term memory that refers to recall of facts, rather than experiences or skills. Be careful not to confuse semantic memory with semantic networks, choice (D) which are the associations of simular concepts in the mind to aid in their retrieval.

Which component of the nervous system is NOT involved in the initial reflexive response to pain? A. Spinal cord B. Cerebral Cortex C. Interneuron D. Motor neuron

B. the cerebral cortex is not involved in the initial reflexive response to pain. Instead, the sensory receptors send information to the interneurons in the spinal cord, which stimulate a motor neuron to allow quick withdrawal. While the brain does ultimately get the signal, the reflexive withdrawal has already occurred by that time.

Which of the following would be most likely to be stored in long-term memory? A. A list of nonsense words B. A list of the dates of birth of 15 randomly selected people. C. A list of the names of musicians in an individuals favorite bands D. A list of the dates of battles in the Peloponnesian War

C. The self-reference effect indicates that information that is most meaningful to an individual is the most likely to be memorized. Choice (C) is the most personally relevant to the individual memorizing the list.

Which of the following is true of controlled processing? A. It is the means through which information enters short-term memory. B. Information that requires controlled processing cannot become automatic C. It always requires active attention to the information being encoded. D. Most information we can later recall is encoded using controlled processing.

C. This is the definition of controlled processing and is the only answer choice that is necessarily true of controlled processing. Effortful processing is used to create long-term memories, and-with practice-can become automatic, invalidating choices (A) and (B). Most of our day-to-day activities are processed automatically, making choice (D) incorrect.

In Alzheimer disease, a protein called the amyloid precursor protein (APP) is cleaved to form a protein called Beta-amyloid. This protein has a Beta-pleated sheet structure and precipitates to from plaques in the brain. This mechanism of disease is most similar to which of the following? A bacteria B. Viruses C. Prions D. Viroids

C. Prions are infectious proteins that cause misfolding of other proteins. Primarily, prions cause a shift toward beta-pleated sheet confirmations, which leads to decreased solubility and degradability of proteins, ultimately leading to disease. This mechanism is very similar to the one described here for Alzheimer's disease, making choice (C) the correct answer.

Which of the following is an example of a circumstance that could cause a state-dependent recall effect? I. The individual is outside on a rainy day II. The individual is high on marajuana III. The individual is in a manic episode A. I only B. III only C. II and III only D. I, II, and III

C. State-dependent recall is concerned with the internal rather than external states of the individual. As such, both statements II and III are examples of state-dependent circumstances, while statement I might cause a context effect instead.

The biology of emotion involves all the following brain regions EXCEPT the: A. amygdala B. prefrontal cortex C. basal ganglia D. thalamus

C. The amygdala, prefrontal cortex, and thalamus all play a role in the experience of emotions. The basal ganglia are involved in smooth movement and are not primarily responsible for the experience of emotions.

A weight lifter is just able to tell the difference between 100 and 125 pounds. According to Weber's law, the lifter would notice a difference between: A. 125 and 150 pounds B. 5 and 6 pounds C. 25 and 35 pounds D. 225 and 275 pounds

C. Weber's law posits that thresholds are proportional. Going from 100 to 125 is a 25 percent increase. Choice (C) is a 40 percent increase while all the rest are all under 25 percent.

According to Erikson's stage of psycho-social developed, which of the following would be the most important for a recent college graduate to accept? A. Figuring out what identities are most important to him or her B. Feeling like a contributing member of society C. Forming an intimate relationship with a significant other D. Finding a feeling of accomplish in his or her life.

C. As a post-adolescent young adult, this person would be be described by Erikson as experiencing the conflict of intimacy vs. isolation, and so forming significant relationships with other would be a primary goal. Choices (B) and (D) represent the next two stages in life (generativity vs. stagnation and integrity vs. despair, respectively), while choice (A) is the conflict that Erikson would say should have been resolved in adolescence (identity vs. role confusion).

Increase in which of the following hormones causes sleepiness? A. Cortisol B. Growth hormone C. Melatonin D. Oxytocin

C. As light diminishes throughout the day, the pineal gland increases secretion of melatonin, resulting in sleepiness. Cortisol levels, choice (A), increase throughout early morning, resulting in wakefulness. Growth hormone secretion, choice (B), peaks during slow-wave sleep. Oxytocin, choice (D), is associated with uterine contractions in childbirth, milk letdown, and bonding behavior.

Resistance to antibiotics is a well-recognized medical problem. Which mechanism(s) can account for a bacterium's ability to increase its genetic variability and thus adapt itself to resist different antibiotics? I. Binary Fission II. Conjugation III. Transduction A. I and II only B. I and III only C. II and III only D. I, II, and III

C. Bacterial cells reproduce by binary fission, an asexual process in which the progeny is incidental to the parent. Therefore, statement I does not increase genetic variability. Conjegation can be described as asexual mating in bacteria; it is the transfer of genetic material between two bacteria that are temporally joined. Transduction occurs when fragments of the bacterial chromosome accidentally become packaged into viral progeny produced during a viral infection and can subsequently be introduced into another bacterium by the viral vector. Therefore, both statements II and III increase bacterial genetic variability.

A credit card company charges a fee for a late payment. This is an example of: A. positive reinforcement B. negative reinforcement C. positive punishment D. negative punishment

C. Because the credit card company wishes to decrease the behavior of a late payment, this is a punishment, so we can eliminate choices (A) and (B). The company is adding something unpleasant, making this an example of positive punishment.

Which theory of dreaming states that dreams and thoughts during wakeful periods use the same stream-of-consciousness system? A. Activation-synthesis theory B. Problem-solving theory C. Cognitive process theory D. Neurocognitive theory

C. Cognitive theorists proposed in the cognitive process dream theory that wakeful and dreaming states use the same mental system within the brain, particularly stream-of-consciousness. The activation-synthesis theory, choice (A), states that dreams are caused by widespread, random activation of neural circuitry. The problem-solving dream model, choice (B), indicates that dreams are used to solve problems while sleeping due to untethereing of dreams from obstacles perceived while awake. The neurocognitive theorists, choice (D), seek to unify cognitive and biological perspectives by correlating the subjective dream experience with the physiological experience of dreaming.

Which of the following best describes the difference between endolymph and perilymph? A. Endolymph is found in the vestibule. while perilymph is found in the cochlea B. Endolymph is found in the cochlea, while the perilymph is found in the vestibule. C. Endolymph is found in the membranous labyrinth, while perilymph is found in the bony labyrinth. D. Endolymph is found in the bony labyrinth while the perilymph is found in the membranous labyrinth.

C. Endolymph is the potassium-rich fluid that bathes the hair cells of the inner ear, all of which ar found within the membranous labyrunth. Perilymph is found in the space between the membranous labyrinth and the bony labyrinth. Both the membranous labyrinth and bony labyruth contribure to the cochlea and the vestibule, eliminating choices (A) and (B).

A rat is trained to press a lever to obtain food under a fixed-interval schedule. Which of the following behaviors would the rat most likely exhibit? a. Pressing the lever continuously whenever it is hungry. b. Pressing the lever exactly once and waiting for the food pellet before pressing it again. c. Pressing the lever slowly at first, but with increasing frequency as the end of the interval approaches. d. None of the above; the association formed by fixed-interval schedules is too weak to increase behavior.

C. In a fixed-interval schedule, the desired behavior is rewarded the first time it is exhibited after the fixed interval has elapsed. Both fixed-interval and fixed-ratio schedules tend to show this phenomenon: almost no response immediately after the reward is given, but the behavior increases as the rat gets close to receiving the reward.

Which of the following neurotransmitters is associated with both schizophrenia and Parkinson's disease? A. GABA B. Serotonin C. Dopamine D. Enkephalins

C. Schizophrenia is associated with high levels of dopamine or high sensitivity to dopamine. Parkinson's disease is associated with destruction of the dopaminergic neurons in the basal ganglia.

Language consists of multiple components. Which of the following involves the order in which words are put together? A. Phonology B. Semantics C. Syntax D. Pragmatics

C. Syntax referes to how words are put together to form sentences and create meaning. Phonology, choice (A), refers to the actual sounds of language. Semantics, choice (B), refers to the association of meaning with a word. Pragmatics, choice (D), refers to change in usage, wording, and inflection based on context.

A student is volunteering in a hospital with a stroke center. When asked what he believes is the prevalence of stroke among those greater than 65 years old, the student states that it is probably about 40% even though data analysis indicates that it is significantly lower. What accounts for this error? A. Deductive reasoning B. Representativeness heuristic C. Base rate fallacy D. Confirmation bias

C. The base rate fallacy occurs when prototypical or stereotypical factors are used for analysis rather than actual data. Because the student is volunteering in a hospital with a stroke center, he sees more patients which have experienced a stroke than would be expected in a hospital without a stroke center. Thus, this experience changes his perception and results in base rate fallacy. Deductive reasoning, choice (A), refers to drawing conclusions by integrating different pieces of evidence. The representative hueristic, choice (B), involves categorization based on how well an individual example fits its category. . Confirmation bias, choice (D), occurs when a person only seeks information that reinforces his or her opinions.

A child has experienced nervous system damage and can no longer coordinate the movements to dribble a basketball, although she can still walk in an uncoordinated fashion. Which region of the central nervous system was most likely affected? A. Forebrain B. Midbrain C. Hindbrain D. Spinal Cord

C. The hindbrain is responsible for balance and motor coordination, which would be necessary for dribbling a basketball. The midbrain, choice (B), manages sensorimotor reflexes that also promote survival. The forbrain, choice (A), is associated with emotion, memory, and higher-order cognition. The spinal cord, choice (D), is likely not damaged as the child can walk.

Which part of the brain deals with both homoeostasis and emotions? A. Cerebellum B. Pons C. Hypothalamus D. Thalamus

C. The hypothalamus is responsible for homeostatic and emotional functions. The cerebellum, choice (A), is responsible for maintaining posture and balance while the pons, choice (B), is above the medulla and contains sensory and motor tracts between the cortex and the medulla. The thalamus, choice (D), acts as a relay station for sensory information.

A person proofreading a paper reads over a long, misspelled word in which an "e" is replaced with an "o." The person does not recognize the error and reads the word as correct. Which of the following could explain why the proofreader read the word as correct? A. Parallel processing B. Feature detection C. Top-down processing D. Bottom-up processing

C. The proofreader used a larger pattern to identify the word and expected to see an "e", thus missing the error. This is related to top-down processing; the proofreader sued recognition and expectations, which led to missing a detail. Bottom-up processing, choice (D), would be the analysis of each detail individually before creating a cohesive image.

Upon which part of the eye are images projected and transduced into electrical signals? A. Cornea B. Pupil C. Retina D. Lens

C. The retina is the part of the eye upon which images are projected. Robs and cones in the retina then convert the electricomagnetic radiation into electrical signals.

Which of the following of Piaget's stage of cognitive development occur before adolescence? I. Preoperational II. Sensorimotor III. Formal Operational A. I only B. II only C. I and II D. II and III

C. The sensorimotor, preoperational, and concrete operational stages occur prior to adolescence. The formal operational stage generally coincides with adolescence.

An individual memorizes a shopping list by associating each item with an image that corresponds with a number. This individual is using which of the following mnemonics? A. Clustering B. Method of loci C. Elaborative rehearsal D. Peg-words

D. The association of words on a list to a preconstructed set of ideas is common to both the method-of-loci and peg-word mnemonics. Method-of-loci systems choice (B) associate items with locations, while peg-word systems use images associated with numbers.

Which of the following is NOT a function of the smooth endoplasmic reticulum? A. Lipid synthesis B. Posion detoxification C. Protein synthesis D. Transport of proteins

C. The smooth endoplasmic reticulum is involved in the transmission of materials throughout the cell, in lipids synthesis, and in the detoxification of drugs and poisons. Proteins from the rough ER can cross into he smooth ER, where they are secreted into cytoplasmic vesicles and transported to the Golgi apparatus. Thus, from the given choices, protein synthesis is not a function of the smooth ER, but rather of the free ribosomoes or the ribosomes associated with the rough ER. Choice (C) is therefore the correct answer.

Many pets will run toward the kitchen when they hear the sound of a can opener opening a can of pet food. The sound of the can opener is a: a. conditioned response. b. unconditioned response. c. conditioned stimulus. d. unconditioned stimulus.

C. The sound of a can opener would not normally produce a response on its own, making it a stimulus that musty have been conditioned by association with food.

A man feels extremely guilty after having an extramarital affair. According to the psycho-dynamic perspective, which of the following is responsible for this anxiety? a. The id b. The ego c. The superego d. The libido

C. The superego is responsible for moral guilt when we do not live up to our ideals. While the id and the libido, choices (A) and (D), may be responsible for the urge to have an affair, the superego is responsible for the anxiety one feels afterwards.

Experiencing emotion involves three components, which are: A. Behavioral, reactionary, and cognitive. B. emotional, physical, and mental C. physiological, cognitive, and behavioral D. emotional, cognitive, and behavioral

C. The three components of emotion are the physiological (changes in the autonomic nervous system), cognitive (subject interpretation of an emotion), and behavioral (facial expressions and body language) responces

A man is looking for change to do laundry. He decides to look under the seats of his car. He uses a flashlight, but is still unable to get more than an obscured look at the space below. There are various items such as wrappers and papers, but the man sees the glint of silver from an object laying flat and determines it to be a coin. To make this determination, this man used: A. Signal detection B. Sensory abaptation C. Feature detection D. Kinesthetic Sense

C. This man was able to distinguish the coin from other items by recognizing specific features of the coin, in this case, it was the glint of the metal surface and its position in the car. This phenomenon is called feature detection.

Herpes simplex virus (HSV) enters the human body and remains dormant in the nervous system until it produces an outbreak after exposure to heat, radiation, or other stimuli. Which of the following best describes HSV? A. While it remains dormant in the nervous system, the virus is in its lytic cycle B. During an outbreak, the virus is in the lysogenic cycle C. Herpes simplex virus adds its genetic information to the genetic information of the cell. D. the herpes simplex virus contains a tail sheath and tail fibers

C. Viruses can exist in either the lytic or lysogenic cycle; they may even switch between them throughout their lifetime. During the lytic cycle, the virus's DNA takes control of the host cell's genetic machinery, manufacturing numerous progeny. In the end, the host cell bursts (lyses) and releases new virons, each capable of infecting other cells. In the lysogenic cycle, viral DNA is added to the host cell's genome, where it can remain dormant for days or years. Either spontaneously or as a result of environmental circumstances, the provirus can reemerge and enter a lytic cycle. Thus, choices (A) and (B) are incorrect because they reverse part of the cycle the virus is in. Choice (D) describes features of bacteriophages, which are viruses that infect bacteria - not the human nervous system. Choice (C) accurately describes how HSV operates during the lysogenic cycle.

The statement, "I noticed my heart racing and breathing rate increasing when I saw a bear, so I am afraid," corresponds most closely with which theory of emotion? A. Schachter-Singer theory B. Yerkes-Dodson theory C. Cannon-Bard theory D. James-Lange theory

D. Experiencing a physiological reaction to a stimulus and then labeling that response as emotion is in line with the James-Lange theory of emotion. In the statement, seeing the bear is the stimulus, an increase in heart rate, and breathing rate is the physiologic reaction, and identifying this as fear is the emotion experienced.

Which of the following is true regarding nerve cells? A. Sensory neurons are also referred to as efferent neurons B. Interneurons are also referred to as afferent neurons C. motor neurons transmit information from receptors to the brain D. Sensory neurons transmit information from receptors to the brain

D. Sensory neurons are also referred to as afferent neurons, whole motor neurons are also referred to as efferent neurons, eliminating choices (A) and (B). Motor neurons transmit motor information from the brain to the body, contrary to choice (C), and sensory neurons transmit sensory information from receptors to the brain.

A patient comes in with a tumor of the pituitary gland, which grows upward into the optic chiasm and causes a visual field defect. The most likely defect from compression of the optic chiasm is: A. Complete blindness in one eye B. Loss of the upper visual fields of both eyes C. loss of the nasal visual fields in both eyes D. loss of the temporal visual fields in both eyes

D. The optic chiasm houses the crossing fibers from each optic nerve. Specifically, the fibers coming from the nasal half of the retina in each eye cross in the chiasm to join the optic tract on the opposite side. Remember that the lens of the eye causes inversion, so images on the nasal half or the retina originate in the temporal vision field. This condition is called bitemporal hemianopsia.

During which of the following stages does dreaming occur? I. Stage 3 II. Stage 4 III. REM A. I only B. II only C. III only D. I, II, and III

D. About 75% of dreaming occurs during REM sleep, but dreams occur in all other stages of sleep as well. More bizarre dreams are likely to occur during REM sleep.

Which of the following neurotransmitters is NOT classified as a catecholamine? A. Epinephrine B. Norepinephrine C. Dopamine D. Acetylcholine

D. Acetylcholine is not a catecholamine; however epinephrine, norepinephrine, and dopamine are.

Which of the following is a conclusion that can be made from research in role-taking and observational learning? A. Young children will only model actions performed by their parents. B. celebrities and athletes are an adolescent's most important role models C. Children who role-take identities that are not gender typical are more likely to take on those roles later in life. D. A female child is more likely to model the behavior of another female than a male.

D. Choice (C) has no support from role-taking research and can be eliminated. Choices (A) and (B) are both inaccurate; the Bobo doll experiments shows young children modeling behavior not performed by their parents, and teens are most likely influenced by their peers, not celebrities and athletes. The research does not suggest, however, that children are more likely to engage in behavior modeled by individuals who are like themselves; thus, a female child is more likely to imitate behavior by another female.

A corporate logo uses five unconnected angles equally spaced in a circular fashion. When viewed, it appears to be a star. Which of the following is the logo artist using to create a complete pattern to viewers? A. Bottom-up processing B. Top-down processing C. Gate theory D. Gestalt principles

D. Gestalt Principe are the basis for many optical illusions and include the tendency of people to see continuity even when lines are unconnected. Specifically, this logo appears to rely on the law of closure to create one complete star from five non touching angles.

Which of the following types of nucleic acid could form the genome of a virus? I. Single-stranded RNA II. Double-Stranded DNA III. Single-Stranded DNA A. I only B. II only C. I and II only D. I, II and II

D. In a virus, the nucleic acid can be either DNA or RNA and - in both cases - can be either single or double stranded. Therefore, all of the types of nucleic acids list here could be used for a viral genome, making choice (D) the correct answer.

Which of the following terms describes how existing schemata are modified to incorporate new information? A. Assimilation B. Adaptation C. Affirmation D. Accommodation

D. Jean Piaget hypothesized that new information is processed by adaptation, choice (B). Adaptation is too broad of an answer because it includes both assimilation, choice (A), and accommodation, choice (D). Assimilation is incorporation of new information into existing schemata. If new information doesn't fit, then accommodation occurs. Accommodation is the modification of existing schemata to account for new information an is thus the correct answer.

Hyperbaric O2 my be used as a treatment for certain types of bacterial infections. In this therapy, the patient is placed in a chamber in which the partial pressure of O2 is significantly increased, increasing the partial pressure of O2 in the patient's tissues. This treatment is most likely used for infections with A. Obligate aerobic bacteria B. facultative anaerobic bacteria C. aerotolerant anaerobic bacteria D. obligate anaerobic bacteria

D. Obligate anaerobes cannot survive in the presence of O2 and would likely be killed by such a therapy, treating the infection. The other types of bacteria listed can all survive in the presence of O2 and would likely not be treated by using this therapy.

Simultaneous processing of conscious emotions and physciological activation is the defining feature of which theory of emotion? A. Schachter-Singer theory B. James-Lange theory C. Incentive theory D. Cannon-Bird theory

D. The Cannon-Bard theory of emotion is based on the premise that conscious feeling and physiological components of emotion are experienced at exactly the same time. In this theory, this combination then leads to action. This is commonly confused with the Schachter-Singer theory, choice (A), in which nervous system arousal occurs and then is labeled based on the context provided by the environment.

When practicing her recital song at home, a teenage girl sounds perfectly in pitch to her family and friends. However, when performing at the recital in front of a large audience of peers, strangers, and coaches, her pitch and tone are off resulting in a poor performance. This second performance is best explained by: A. drive reduction theory B. instinct approach theory C. Maslow's hierarchy of needs D. the Yerkes-Dodson Law

D. The Yerkes-Dodson law states that there is an optimal level of arousal necessary to perform. If levels of arousal are too high, poor performance can result. In the case of this girl performing at her recital, her arousal level is very high as a result of nervousness and anxiety, resulting in a poor performance.

During a physical examination, a physician brushes the bottom of the foot of a fifty-year-old patient with multiple sclerosis. Her toes are observed to curl toward the bottom of her foot, with no fanning of the toes. This response is: A. Abnormal, and evidence that she is exhibiting a primitive reflex B. Normal, and evidence that she is exhibiting a primitive reflex C. Abnormal, and evidence that she is not exhibiting a primitive reflex D. Normal, and evidence that she is not exhibiting a primitive reflex

D. The babinski reflex is a premitive reflex that referes to an extension of te big toe accompanied by fanning of the other toes. It is normal in infants, but should disappear with time - certainlly by the time a child begins to walk. In a fifty-year-old-woman, the babinski reflex would be abnormal. However, despite her neurological illness, this patient is exhibiting a normal responce to the brushing of her foot; that is, she is not showing babinski reflex.

A 19-year old college student is picked up by campus police after shoplifting a large bag of corn chips and a dozen ice cream sandwiches. His eyes are bloodshot. During questioning, he repeatedly asks for water because his mouth is dry, and he cannot stop giggling. What is the psychoactive substance in the drug this student has most likely recently taken? A. Alprazolam B. 3,4-methylenedioxy-N-methylamphetamine C. Diacetylmorphine D. Tetrahydrocannabinol

D. The description of the student matches the clinical features of marijuana (cannabis) use; hunger (presumably, based on his loot), redness of the eyes, dry mouth, and euphoria. Marijuana may also cause an increase in heart rate, short-term memory loss, paranoia, and-in high doses- hallucinations. Tetrahydrocannabinol is the primary substance in marijuana.

A man is at a party with his wife. There is loud music in the background and the location is crowded. While listening to the music he hears what he believes to be his wife's laughter and turns around to investigate. The man is exhibiting: A. feature detection B. bottom-up processing C. vestibular sense D. signal detection

D. The man is discerning a specific noise within a field of many noises. This is the definition of signal detection. In an experimental setup, his response would be considered a hit if his wife was indeed laughing; his response would be considered a false alarm if his wife was not laughing.

The temporal lobe deals with all of the following EXCEPT: A. Language comprehension B. Memory C. Emotion D. Motor Skills

D. The temporal labes have many functions, but motor skills are not associated with this area. The temporal lobes contain Wernicke's area, which is responsible for language comprehension. choice (A). The temporal lobes also function in emotion and memory, choices (B) and (C), because they contain the amygdala and hippocampus. Motor skills are associated with the frontal lobe (primary motor cortex), basal ganglia (smooth movements), and cerebellum (coordination).

A child finds that she cannot make an origami swan by herself, but is able to do so when observing and being assisted by an adult. This scenario is described in the ideas of which of the following theorists? a. Albert Bandura b. Alfred Adler c. B.F. Skinner d. Lev Vygotsky

D. This situation is best described by Lev Vygotsky's zone of proximal development theory, which holds that children are often unable to perform tasks by themselves, but can be complete task with the help of a more knowledgeable other.

Which of the following processes would increase the likelihood of a behavior? A. extinction B. negative punishment C. positive punishment D. avoidance learning

D. Avoidance learning is a type of negative reinforcement in which behavior is increased to prevent an unpleasant future consequence. Extinction, choice (A), is a decreased response to a conditioned stimulus when it is no longer paired with an unconditioned stimulus. Punishment, choices (B) and (C), lead to decreased behaviors in operant conditioning.

Determination of the intensity and risk of a stressor occurs during which stage(s) of stress appraisal? A. Primary appraisal only B. Secondary appraisal only C. both primary and secondary appraisal D. neither primary nor secondary appraisal

Secondary appraisal of stress is the stage at which the ability of the organism to cope with the stressor is evaluated. This is based on the harm, threat, and challenge of the stressor, which are all correlated with it's intensity. Primary appraisal is Simpy the initial determination of whether there is a negative association at all, not it's intensity.

A person with high left frontal lobe activity is most likely experiencing which emotion? A. Happiness B. Sadness C. Surprise D. Disgust

The left frontal lobe is assosiated with positive feelings, coresponding with joy and happiness. The right frontal lobe is assosiated with negative feelings, such as sadness and disgust, choices (B) and (D).

A nine-year-old girl is brought to the pediatrician. Her parents describe that any time she is startled, she appears to collapse and fall asleep. She also complains of waking up in the morning unable to move. Which sleep disorder should be suspected? a. Insomnia b. Sleep deprivation c. Narcolepsy d. Sleep apnea

The patient and her parents are describing cataplexy ( a sudden loss of muscle tone and intrusion of REM sleep during waking hours, usually in responce to a startling or emotional trigger) and sleep paralysis (an inability to move despite being awake, usually when waking up in the morning). These symptoms are highly suggestive of narcolepsy; in fact, some consider cataplexy to be pathonomic for (absolutely indicative of) the disorder.


Ensembles d'études connexes

Sport Management: Test 1 (Chapters 1-5)

View Set

State and Local Government Final

View Set

Drugs for Angina Pectoris & Management of STEMI

View Set

Live Virtual Machine Lab 2.1: Module 02 Defining Networking Devices

View Set